Sunteți pe pagina 1din 32

Contents Page

No.
From the Directors Desk: Divide your time focus on each section 3
Victory Story
Be honest to yourself during preparation Livie Jain 4
Current Affairs Timeline
National News 5
International News 6
Banking and Economy 8
Appointment & Resignations 8
Awards and Recognitions 10
Obituaries 11
Sports News 12
Defence and Space 14
Science and Technology 14
Current Legal Knowledge
Legal Affairs 14
Legal News 15
LE Explains
Gaganyaan Mission 16
President s Rule in Jammu & Kashmir 17
103rd Constitutional Amendment Act 2018 18

JANUARY 2019 20 Legal One Liners


Potpourri
19
Maxims and Foreign Phrases 19
Key Differences Between Important Concepts 20
LE Prep Assist
Mentor View English Language 21
Mentor View Mathematics 22
Prep Test Legal Aptitude 23
Prep Test Mathematics 33
Current Affairs Practice Sheet CAPS
Current Affairs Practice Sheet (CAPS) 35
Answer Key with Explanations
Answer Key with Explanations Prep Test 47
Answer Key with Explanations (CAPS) 51

Published by: LegalEdge Tutorials. 127, Second Floor, Zone-II, M.P. Nagar, Bhopal 462011
Contact No: 9111555433
www.legaledge.in

©LegalEdge Tutorials Page 1 of 60 ©LegalEdge Tutorials Page 2 of 60


Replication or other unauthorized use of this material is prohibited by the copyright laws of India Replication or other unauthorized use of this material is prohibited by the copyright laws of India
Desk: Divide your time; focus on each section “Be honest to yourself during preparation : Livie Jain
work upon that area. I used to revise my mock
Future Lawyers ,
analysis copy before every mock. Also, I used to mark
With roughly 3 months to go for CLAT and AILET, things have started to heat up. A quick reminder, AILET has always the questions that were tricky so as to make it easier
been a lot tougher than CLAT and owing to the introduction of negative marking last year, it may have gotten a bit more to find while revising. I wrote all the formulae at one
fastidious. SET, rechristened SLAT, since 2018, has also gone online, These changes, on their face might seem as an added place and read them again and again.
layer of difficulty but are actually good in that they make the exams more deliberate and minimize "fluke-success." 4. Were you scared of any subjects in the beginning?
What did you do to overcome that?
So, what can we expect from every LegalEdge student in the last 3 months? I always thought the most boring thing is to read a
1. You have to divide your time between the chapters you still haven't completed and the ones you need to revise. newspaper and the result was me being the worst in
Perfecting control over the existing territories is as important as conquering new ones. general awareness. The aversion to GK persisted for a
long time as I was able to manage to score good with
2. GK sections in past few years have been tilted towards Current Affairs. That the same will continue in 2019 is not the help of other subjects. But soon the teachers made
Livie Jain (AILET AIR 11) me realize that I cannot depend upon any one section
entirely certain. Therefore, do keep the Static GK also in check. The only way to remember the news is to read, re-
(NLU DELHI) and at the same time I cannot ignore any particular
read and keep revising. You must take an hour out for Current Affairs (30mins for new and another 30mins for 1. How instrumental has LegalEdge been to you for
revising) and 30mins for Static GK on a daily basis. section. Then I started reading and revising. I devoted
your preparation for CLAT? a particular time period for studying GK. No matter
Taking a drop was one of the best decisions that I ve which test comes up tomorrow, I ll stud GK in the
3. Solving all Past Years Papers of CLAT and AILET is imperative. You will be surprised to know the variety of topics made. Just like any other student, I too was scared and designated hour. This way I had no excuses for myself
and questions covered across all sections. The chapters that you consider unimportant have played crucial roles in anxious about it, but LegalEdge helped me overcome to move away from studying GK. Karan sir helped me
final selection on many occasions. So make sure every chapter and topic is accounted for. this fear. It was not just a coaching centre but a whole a lot in coming up with this strategy. Also, when I
new world for me. Whenever I faced challenges, I used to ask questions from Abhishek Sir, I was always
4. Remember, Legal Reasoning and GK will determine your selections in the top Law Schools of the country but found the faculty ready to help me. There were times fascinated by the plethora of knowledge he had. I
English, Mathematics and Logical Reasoning will determine your ranks in CLAT. Hence, do not make the mistake when I was depressed and on the verge of giving up. really wanted that I also become the one who knows
of taking unreasonable favourites. Focus on each section, having understood its specific importance. But the teachers taught me how this is all a part of the each topic that I come across thoroughly. This
process. All the pain was worth bearing. After all, I motivated me to read more and more. And once you
Understandably, you are facing the most demanding of times in terms of the time, energy and effort that you have to was taking shape, I was growing. This attitude of start loving something, it s no longer a burden for ou,
invest and the surmounting anxiety that you experience. Just remember that while it is you and only you who can dream never giving up is the best gift of LE. it comes naturally.
and go on to realise it, you will always have us to share your load with, understand your needs and help you at each step, 2. How did you prepare for AILET? Was your approach 5. How did you analyze your mocks?
different or same for both CLAT and AILET?
as always! Mock analysis is the only thing that the mocks are
According to me, both the exams require a different
made for. There is nothing to be worried about the
approach when it comes to attempting the paper. The
Regards, scores. In the end they only tell you which section
preparation part is almost the same. AILET requires a
needs to be worked upon. I used to spend 4 to 5 hours
firm hand on English, good reading speed and a good
in analyzing my mocks. The first reading was to find
judgment because of the length of the paper. You must
out how I have performed and to identify the flukes.
be prepared for the fact that you cannot attempt all the
The next reading was for the wrong and skipped
questions and there will be a section that has to be
questions and for the flukes. I used to write them all
compromised. But don t panic. I m sure if I had
in m notebook. Then I marked the questions that I m
panicked then the result would not have been the
unable to solve and cleared the doubts later. I never
same. It was the end time decision of switching the
left any question unasked if it is bothering me even a
section that proved to be pivotal.
little bit. Getting to know the reasoning behind every
So, all in all it depends more upon how you attempt
answer was my approach rather than solving
the paper than how you prepare for it. And of course,
particular questions.
there cannot be any section that you ignore 6. Who was your inspiration throughout your
completely during preparation. Presence of mind is preparation?
the key. I wanted to get into one of the top law colleges in the
3. What was your revision strategy while preparing for country and that was what I kept reminding myself.
CLAT?
Whenever I thought I would stop working, my
Revise and revise and revise The more number of
mother used to motivate me to work a little bit more.
times you revise, the better. Maintaining a mock
She was sure I will do it. That kept the fire burning.
analysis copy helped me in this. I used to solve/write
Also the teachers at LE are so optimistic they will
all the questions that confused me during the test and
never let you stop. I remember every time I was not
all the questions that were answered wrong. It
feeling well, Harsh sir filled me with a new energy.
Harsh Gagrani Karan Mehta becomes important to be honest to yourself during
Once you start speaking to people who motivate you,
(Director, LegalEdge) (Director, LegalEdge) preparation. If you lie to yourself, you are destroying
it becomes like a drug that keeps you alive. Never
your future. You must remember all the flukes and
stop, never give up, keep learning.

©LegalEdge Tutorials Page 3 of 60 ©LegalEdge Tutorials Page 4 of 60


Replication or other unauthorized use of this material is prohibited by the copyright laws of India Replication or other unauthorized use of this material is prohibited by the copyright laws of India
Current affairs Timeline Current affairs Timeline
officials remained at the site until the goal was Nicobar Islands, across the international waters.
NATIONAL NEWS World s Youngest Mountaineer To Climb 7 Peaks, completed. Pabuk is expected move west-northwestwards
India s 70th Republic Da parade themed on 7 Volcano Summits Citizenship Amendment Bill 2019 Passed in Lok and emerge into the Andaman Sea.
Mahatma Gandhi Mountaineer Satyarup Siddhanta conquered Sabha JNPT Becomes Only Indian Port To Be Listed
South African President Cyril Ramaphosa attended Antartica s highest point, Mount Sidley. With this The Citizenship Amendment bill seeking to Amongst World s Top 30 Container Ports
Republic Day celebrations in the national capital as feat, he became the first Indian and youngest man provide Indian citizenship to persecuted The Jawaharlal Nehru Port Trust (JNPT), India s
the Chief Guest on Saturda . This ear s Republic to climb the 7 highest peaks and volcanic summits minorities from Bangladesh, Pakistan, and premier container port got listed amongst the top
Day celebrations were themed on the 150th birth across all continents. He achieved the feat at the age Afghanistan was passed by Lok Sabha. The Bill 30 container ports globally, as per the latest Lloyds
anniversary of Mahatma Gandhi. of 35 years and 262 days. Satyarup hails from South seeks to amend the Citizenship Act of 1955 in order Report. JNPT notched up five spots, to be 28th on
Microsoft Launches e-Commerce Platform for Kolkata and is a software engineer by profession. to grant citizenship to Hindus, Buddhists, Sikhs, the list, compared to its previous ranking.
Handloom Weavers Jains, Parsis and Christians from Bangladesh, Arunachal Government Declares State Festival
Microsoft India has launched a new e-commerce Afghanistan and Pakistan if they have lived in Arunachal Pradesh Chief Minister, Pema Khandu
platform re-weave.in for handloom weavers under India for six years. declared Pakke Paga Hornbill Festival (PPHF) as
its Project ReWeave, part of its philanthropic 6 Monuments Declared Of National Importance the State Festival at Seijosa in East Kameng
initiatives. This e-commerce platform will help In 2018 By ASI district, Arunachal Pradesh. Pakke Paga Hornbill
connect artisans to buyers directly, enabling them These sites are:125-year-old Old High Court Festival (PPHF) is only conservation festival of
to expand to newer customers and markets. Building in Nagpur, Maharashtra, The Group of Arunachal Pradesh.
First State To Implement 10% Quota For General Temples in Ranipur Jharail in Bolangir district of
Category Odisha,2 Mughal era monuments in Agra- Haveli
Gujarat has become the first state to implement of Agha Khan and Hathi Khana, The Neemrana
the 10% reservation for economically weaker Baori in Alwar district of Rajasthan, The Vishnu
sections of the general category. This comes a day Temple in Kotali, Pithoragarh district in
after President Ram Nath Kovind gave his assent to Uttarakhand.
the bill. ACI s Ease Of Doing Business Index
National Youth Day Andhra Pradesh emerged on top of Asia
The National Youth Day is celebrated across the Competitiveness Institute (ACI), Singapore s 2018
India Among The Most Trusted Nations India Ranks 80th On Global Talent
country on 12th January. NYD 2019 is the 156th Ease of Doing Business (EDB) Index ABC rankings Competitiveness Ranking
Globally: Report for economies of 21 states of India, while
India is among the most trusted nations globally birth anniversary of Swami Vivekananda. Since India has moved up one position to rank 80th on
1985, his birth anniversary has been celebrated as Maharashtra and Delhi came in second and third the global talent competitive index, but remains a
when it comes to government, business, NGOs and place, respectively.
media. The 2019 Edelman Trust Barometer report, National Youth Day. Vivekananda died at the laggard among the BRICS nations. Switzerland
young age of 39. World s 1st Female Amputee To Scale continues to top the list, released on the first day of
noted that the Global Trust Index witnessed a Antarctica s Highest Peak
India s Longest Single Lane Steel Cable Bridge the World Economic Forum (WEF) Annual
marginal increase of 3 points to 52. China topped Arunima Sinha, the first female amputee to have
India s longest 300-meter single lane steel cable Meeting at Davos by INSEAD business school in
the Trust Index among both the informed public conquered Mt Everest, created yet another record
suspension bridge was inaugurated by Arunachal partnership with Tata Communications and
and the general population segments. India was at as she became world s first woman amputee to
Pradesh Chief Minister Pema Khandu. The bridge Adecco Group.
the 2nd place in the informed public category and climb Mount Vinson, the highest peak in
across the Siang river at Yingkiong in Upper Siang National Girl Child Day
3rd place in the general population category. Antarctica. Apart from Mount Everest, the 30-year-
district has been named Byorung Bridge. It is built National Girl Child Day is celebrated in India on 24
India To Surpass UK in World s Largest Econom old has climbed the highest peaks of five continents
Rankings in 2019 at a cost of `48.43-crore provided by the DONER January. Women and Child Development Ministry
(Department for Development of North Eastern and plans to do so in all seven. celebrated the National Girl Child Day 2019 at
India is likely to surpass the United Kingdom in the
world s largest econom rankings in 2019, Region) Ministry. Pravasi Bharti Kendra, New Delhi. The theme of
according to a report by global consultancy firm National Girl Child Da 2019 was Empowering
PwC. According to World Bank data, India became Girls for a Brighter Tomorrow .
the world s sixth largest econom in 2017
surpassing France and was likely to go past the UK
which stood at the fifth position. INTERNATIONAL NEWS
15th Pravasi Bhartiya Divas Begins
Malala Pens New Book On Refugee Girls
The 15th edition of Pravasi Bhartiya Divas begins
Nobel Peace Prize winner and Pakistani activist
at Varanasi in Uttar Pradesh. The 3 day event is Polavaram Project Enters Guinness Book of Malala Yousaf ai pens a new book, titled We Are
being organised in Varanasi for the first time. The World Record Displaced: My Journey and Stories from Refugee
theme this ear is, Role of Indian Diaspora in The Polavaram project of Andhra Pradesh entered Girls Around the World, charting her experiences
building New India . Several world leaders of the Guinness Book of World by pouring 32,100 traveling the world and visiting refugee camps.
C clone Pabuk to enter Andaman Sea
Indian origin, including the Chief Guest Mr. cubic meters of concrete in a matter of 24 hours. India Meteorological Department (IMD) has
Pravind Kumar Jugnauth, Prime Minister of While the staff of Navayuga Engineering company located the c clonic storm Pabuk over the South
Mauritius, will take part in the event and share performed the task, project, and state government China Sea. It was seen about 1,500 km east-
their insights on the theme.
southeast of Port Blair, capital of Andaman and

©LegalEdge Tutorials Page 5 of 60 ©LegalEdge Tutorials Page 6 of 60


Replication or other unauthorized use of this material is prohibited by the copyright laws of India Replication or other unauthorized use of this material is prohibited by the copyright laws of India
Current affairs Timeline Current affairs Timeline
World Hindi Day is celebrated on January 10 every Qatar Withdraws From OPEC
year, marking the anniversary of first World Hindi Qatar has withdrawn its membership from
Conference which was held in 1975. World Hindi Organization of the Petroleum Exporting Countries
Day and National Hindi Diwas are completely (OPEC). The country had sent an official
different. National Hindi Diwas is celebrated every notification to the OPEC expressing its wish to pull
year on September 14. out and to focus on its liquefied natural gas
2019 African Cup of Nations production in December. Qatar has been in OPEC
Seven-time champions Egypt have won the for 57 years. The decision came amid the ongoing
World Economic Forum recognises Tata Steel s
hosting rights for the 2019 edition of the African diplomatic and economic blockade imposed on
Netherlands plant as factor of the future
Cup of Nations. Egypt beat South Africa by 16 the country by its Persian Gulf neighbours and
The World Economic Forum on Thursday
votes to one at the Confederation of African several Arab states.
announced that Tata Steel s plant at Ijmuiden in
Football s (CAF) Executive Committee. With just
the Netherlands has been inducted into its
1st World Capital of Architecture for 2020
five more months for the tournament, Egypt has ECONOMY AND BANKING prestigious community, a distinction awarded to
got a limited time to prepare for an event where a Fugitive Economic Offender
The United Nations Educational, Scientific, and manufacturing facilities which are seen as leaders
total of 24 teams will be participating. Vijay Mallya became the first tycoon to be charged
Cultural Organization (UNESCO) has named the in technologies of the Fourth Industrial
Europe dominates top of democracy index, North under a new anti-fraud law, with a court in
Brazilian city of Rio de Janeiro as the World Revolution .
Korea lags in last Mumbai naming him as an offender under the
Capital of Architecture for 2020. As the first World India To Become World s 3rd Largest Consumer
The Economist Intelligence Unit (EIU) has Fugitive Economic Offenders Act, 2018. The anti- Market By 2030
Capital of Architecture, Rio de Janeiro will hold a
released its annual democracy index, in which corruption court was hearing an application by the A report by World Economic Forum stated that
series of events under the theme All the worlds.
Australia and New Zealand are the onl full Enforcement Directorate, seeking a direction to India is set to become the world s third largest
Just one world, and promote the internationally
democracies in the entire Asia-Pacific region, declare Mr. Mallya a fugitive economic offender. consumer market behind only USA and China by
agreed 2030 Agenda for Sustainable
while the United States is among those that 2030. The report was titled as Future of
Development s 11th Goal: Make cities and human
couldn t find its wa into the top categor . There Consumption in Fast-Growth Consumer Market
settlements inclusive, safe, resilient and
are also eight countries (Algeria, Democratic India . As per the report, consumer spending in
sustainable.
Republic of Congo, Timor-Leste, Ethiopia, North India is expected to grow from 1.5 trillion US dollar
Korea, Laos, Nepal and Sri Lanka) whose full titles at present to six trillion US dollar by 2030.
include the word democratic , but the EIU sa s
not one of these countries is actually fully APPOINTMENTS AND
democratic. RESIGNATIONS
Most Powerful Passports Of 2019 Andry Rajoelina Sworn In As Madagascar
For the second time in a row, Japan retained its top Nepal s Central bank announces ban of Indian President
spot as the world s most powerful passport in 2019, notes Madagascar president, Andry Rajoelina, has
according to the Henley Passport Index. The Nepal s central bank has banned the use of Indian
WEF Annual Meet Begins sworn into office after taking the oath in
countr became the world s most travel-friendly currency notes of Rs 2,000, Rs 500 and Rs 200
The World Economic Forum (WEF) annual meet Antananarivo before nine judges of the High
passport due to the document s access to 190 denominations, a move that could affect Indian
had begun in Davos, Switzerland. The theme of Constitutional Court. He won nearly 56% to defeat
countries. Singapore retains 2nd Spot; India at tourists visiting the Himalayan nation where
the event would be Globalization 4.0: Shaping a another former president, Marc Ravalomanana.
No.79. Indian currency is widely used. Indian notes of 100
Global Architecture in the Age of the Fourth
Venezuela Breaks Off Diplomatic Ties With The or below, however, are allowed for trading and
Industrial Revolution . Those expected to attend
US conversion.
include German Chancellor Angela Merkel, Swiss
Venezuelan President Nicolas Maduro has GDP To Be A Tad Higher In FY20
President Ueli Maurer, Japan s Shin o Abe, Ital s
announced that he is breaking off diplomatic ties India Ratings and Research (Ind-Ra), a Fitch
Giuseppe Conte and Israel s Benjamin Netan ahu.
with the United States after his American Group Compan , estimated India s GDP growth
Arab Economic And Social Development Summit
counterpart Donald Trump recognized Opposition could touch 7.5 % in the financial year 2019-20 as
The Arab Economic and Social Development
leader Juan Guaido as the South American against 7.2 % during current fiscal i.e. 2018-19.
Summit held in Lebanon s capital Beirut. The
economic meeting is a prelude to the actual Arab countr s interim President. Nicolas Maduro gave India Allows Iran s Bank To Open Branch in
all US diplomatic staff in Venezuela 72 hours to Mumbai
League summit taking place in Tunisia in March.
New York breaks tourism record in 2018 leave the country. The Venezuelan President Union Minister Nitin Gadkari has announced that
India has allowed an Iranian bank, Pasargad Bank, Swedish lawmakers elect PM Lofven to second
The New York tourist board said that a record- accused the US government of attempting a coup term
breaking 65.2 million tourists visited New York in d etat. to open a branch in Mumbai. The move is aimed at
circumventing US trade and investment sanctions Sweden ended a four-month political vacuum on
2018 with British people particularly keen to visit International Day of Education Friday when lawmakers elected Prime Minister
the Big Apple from overseas. According to new 24th January is the International Day of Education. as India builds a strategic port in the West Asian
country. Stefan Lofven to a second term, after he elbowed
figures from NYC & Company, around 51.6 million On 3 December 2018, the United Nations General out the far-right to save one of Europe s few left-
Americans and 13.5 million foreign tourists visited Assembly adopted with consensus a resolution wing governments. It will be one of the weakest
the city last year. Britons topped the list of overseas proclaiming 24 January as International Day of administrations in Sweden in 70 years, with just
visitors, with 1.24 million, followed by China. Education, in celebration of the role of education 32.7 per cent of voters having cast ballots for the
World Hindi Day for peace and development. two parties.
©LegalEdge Tutorials Page 7 of 60 ©LegalEdge Tutorials Page 8 of 60
Replication or other unauthorized use of this material is prohibited by the copyright laws of India Replication or other unauthorized use of this material is prohibited by the copyright laws of India
Current affairs Timeline Current affairs Timeline
Sheikh Hasina Sworn-In As Bangladesh PM right president since a military dictatorship gave Maduro Sworn In As Venezuela President Mohan Malviya were given the award by the
It is her third consecutive term after her victory in way to civilian rule three decades ago. Nicolás Maduro was sworn in for a second term as Narendra Modi government in 2015.
the 11th Parliamentary elections. President Piyush Goyal Named Interim Finance Minister Vene uela s President amid the economic and Ranjani Murali Wins Woman s Voice Award
Mohammad Abdul Hamid administered the oath Piyush Goyal has been named the interim Finance humanitarian crisis faced by the Latin American US Based Indian poet Ranjani Murali was
of office to Sheikh Hasina in Dhaka. This is the first Minister and interim Minister of Corporate country. The US, EU and 13 other countries have awarded the Woman s Voice Award at the
time Hasina is forming a cabinet with the members Affairs during Arun Jaitle s indisposition. Goyal stated that the would not recognise Maduro s Apeejay Kolkata Literary Festival (AKLF). The
of her own party. will retain his existing portfolios of coal and Presidency. Objective of the award is to recognize and
Mala sia s King Resigns railways. Union Minister Arun Jaitley, 66, had left encourage creative writing by women in India. The
Mala sia s King Muhammad V abdicated after for the United States for a regular medical check- Award carried a cash prize of Rs 1 Lakh along with
two years on the throne, the first time a monarch up. Jaitley will be designated as a minister without a citation. Ranjani Murali s second book Clearl
has stepped down before completing their five- a portfolio during his period of indisposition. ou are ESL won the Great Indian Poetr
ear tenure. The king s resignation took effect Mohammed Ali Qamar Appointed Women s Collectives (GIPCs) Editor s Choice award. Her
immediately. A week ago, the king, 49, had Team Coach first book Blind Screens was published in Jul
resumed duties after spending two months on Mohammed Ali Qamar, the man who won India s 2017.
medical leave. first Commonwealth Games gold medal in IWF President Named As Chef de Mission for Sansad Ratna Award
Jim Yong Kim Resigns As President Of The boxing, has taken over as the chief coach for the Tokyo Olympics Himachal Pradesh MP Anurag Thakur has been
World Bank countr s female boxers. He has replaced veteran President of the Indian Weightlifting Federation bestowed with Sansad Ratna Award for his
World Bank President, Jim Yong Kim has made Shiv Singh. Qamar, who is set to turn 38, is also the (IWF), Birendra Prasad Baishya, has been named distinguished performance as a parliamentarian
the surprise announcement that he is stepping oungest person to land the job. Qamar s CWG Chef de Mission for the 2020 Tokyo Olympics. under Jury Committee Special Award category.
down after six years in the post. His resignation gold came in the 2002 Manchester edition in the Baishya is also one of the Indian Olympic Thakur became the first BJP MP to receive the Jury
will take effect from 1 February. Mr. Kim, 59, was light flyweight category. Association Vice Presidents. Committee award. 12 parliamentarians were
not due to leave until 2022 after he was re-elected Harendra Singh Discontinued as Team Coach honored with the Sansad Ratna award, which was
for a second five-year term in 2017. Indian men s hocke coach Harendra Singh was set up in 2010 on the suggestion of ex-President APJ
removed from the position and offered the task of Abdul Kalam.
helming the junior squad by the national Bharat Ratna CNR Rao Chosen For International
federation. Harendra Singh was brought in as Prize For Materials Research
men s coach in Ma 2018 after Sjoerd Marijne was Jawaharlal Nehru Centre for Advanced Scientific
moved back to the women s camp. Research announced that the Centre for Advanced
Pranav R Mehta Becomes 1st Indian to Head Materials of the United Arab Emirates (UAE)
Stephen Constantine Resigns chose Eminent Scientist and Bharat Ratna recipient
Global Solar Council
Indian football team head coach Stephen CNR Rao for the first Sheikh Saud International
A visionary in the field of Solar Energy and the
Constantine has stepped down after the team s Prize for Materials Research. CNR Rao was
chairman of National Solar Energy Federation,
shock 0-1 defeat to Bahrain which led to its exit awarded the highest Civilian award, Bharat Ratna
Pentagon Chief of Staff Resigns Pranav R Mehta, has taken over as the president of
from the Asian Cup. The Anglo-Cypriot coach took in 2014 and presently he is working as the
In the United States, Department of Defence, Chief Global Solar Council (GSC). He has become the
over the reins of the team in 2015 for a 2-year Honorary President of Jawaharlal Nehru Centre for
of Staff, Rear Admiral Kevin Sweeney has first Indian to head the GSC. The Global Solar
tenure. He was given one-year contract extension Advanced Scientific Research.
resigned, a month after Defence Secretary James Council (GSC) was launched on December 6, 2015,
twice. It was his second stint as India coach after
Mattis announced his departure. He is now the following the historic United Nations Climate
serving as the coach in 2002-05.
third senior Pentagon official to announce his Change Conference (UN COP 21).
Su Tseng-chang Appointed PM of Taiwan
resignation since President Donald Trump
Taiwan President Tsai Ing-wen has appointed Su
announced US forces would leave Syria.
Tseng-chang as Prime Minister, during a Cabinet
Justice TBN Radhakrishnan Sworn In As 1st
reshuffle following the ruling Democratic
Chief Justice of Telangana High Court
Progressive Part s heav losses in local elections.
The oath to the office was administered to Justice
Su is a former premier appointed in 2006 by then-
Radhakrishnan by Telangana and Andhra Pradesh Gurinder Singh Khalsa Presented With Rosa
President Chen Shui-bian and was a chairman of
Governor ESL Narasimhan at a ceremony held at Parks Trailblazer Award
Tsai s Democratic Progressive Part for two terms.
Raj Bhavan. With this, the state of Telangana will The Indian-American Sikh Gurinder Singh
Albie Morkel Retires From All Forms Of Cricket
have its first independent high court. Khalsa, 45, who lives in Fishers of Indiana, US was
Jair Bolsonaro Takes Oath
Albie Morkel, the South African all-rounder, AWARDS AND RECOGNITIONS presented with the prestigious 2019 Rosa Park
announced his retirement from all forms of cricket. Bharat Ratna
Jair Bolsonaro was sworn in as Bra il s President Trailblazer award after being selected for
Morkel, who played for close to 20 years of Former President Pranab Mukherjee, Bharatiya
in a ceremony at Bra il s National Congress champions of the diversity by the Indiana Minority
professional cricket, made his first-class debut for Jana Sangh leader Nanaji Deshmukh and singer
Building. Bolsonaro is a former Army captain and Business Magazine. The award has been given for
Easterns against Northerns in the 1999-00 domestic Bhupen Hazarika were on Friday conferred the
admirer of the countr s 1964-1985 military his campaign that forced the authorities in the US
season. Morkel earned a call-up into the South countr s highest civilian award Bharat Ratna. The
dictatorship. Bolsonaro, 63, was a seven-term fringe to change their policy towards turban of the Sikh
African ODI set-up for the series against New Bharat Ratna has been conferred after a gap of four
congressman who rode a wave of anti- community and for his continued demonstration of
Zealand in 2004 after snaring 12 scalps in List A years. Former Prime Minister Atal Bihari Vajpayee
establishment anger to became Bra il s first far- courage and compassion.
games in the 2003-04 season. and founder of Banaras Hindu University Madan
Gandhi Peace Prize Conferred for Last 4 Years

©LegalEdge Tutorials Page 9 of 60 ©LegalEdge Tutorials Page 10 of 60


Replication or other unauthorized use of this material is prohibited by the copyright laws of India Replication or other unauthorized use of this material is prohibited by the copyright laws of India
Current affairs Timeline Current affairs Timeline
The Gandhi Peace Prize for the years 2015, 2016, Fields Medal-Winning Mathematician Passes SPORTS
2017 and 2018 has been conferred on the following: Away Naomi Osaka wins Australian Open final to seal
For 2015: Vivekananda Kendra, Kanyakumari for Sir Michael Atiyah, a British mathematician who back-to-back grand slams
their contribution in Rural Development, received the 1966 Fields Medal, also known as the The 21-year-old Naomi Osaka played brilliantly to
Education , Development of natural resources; For Nobel for Mathematics, passed awa at the age beat the Czech Petra Kvitova to win a dramatic
2016: Jointly to Akshaya Patra Foundation for its of 89. Atiyah was best known for his co- Australian Open final. Her second consecutive
contribution in providing mid-day meals to development of a branch of mathematics called grand slam title ensures she will be the new world
millions of children across India and Sulabh topological K-theory and the Atiyah-Singer index No 1 and on this evidence, she might stay there for
International for its contribution in improving the theorem. some time. She is the first woman to follow her first
condition of sanitation in India and emancipation PM receives Philip Kotler award Veteran Bengali Writer Dibyendu Palit Passes
Prime Minister Narendra Modi on Monday grand slam title by immediately winning the next
of manual scavengers; For 2017: Ekal Abhiyan Away
received the first-ever Philip Kotler Presidential one since Jennifer Capriati in 2001 and the first
Trust for their contribution in providing Education Renowned Bengali writer and Sahitya Akademi
award here. The award focuses on the triple since Serena in 2015 to win two slams in a row.
for Rural and Tribal Children in remote areas pan awardee Dibyendu Palit passes away due to age-
India, Rural Empowerment, Gender and Social bottom-line of people, profit and planet It will be related ailments. He was 79-year-old. Palit was
Equality; For 2018: Shri Yohei Sasakawa for his offered annually to the leader of a nation. conferred the Sahitya Akademi for his novel
contribution in Leprosy Eradication in India and Namita Gokhale Wins Sushila Devi Literature Anubhab in 1998. He was awarded the Bankim
Award Purashkar for Dheu in 1990 and Ananda
across the world.
Dr. Rattan Lal Honoured With 2019 Japan Prize Writer Namita Gokhale has won the Sushila Devi Purashkar in 1984 for Sahojoddha .
The Japan Prize Foundation announced India born Literature Award for her novel Things to Leave Sachin s Coach Ramakant Achrekar Passes Awa
professor Dr. Rattan Lal, University Professor of Behind . She was awarded in the Best Book of Veteran cricket coach Ramakant Achrekar, who
Soil Science at The Ohio State University in Fiction Written b a Woman Author categor at trained Bharat Ratna Sachin Tendulkar and Vinod
the inaugural edition of Bhopal Literature and Art Ankita Raina Wins ITF s Tournament In
Columbus, Ohio, U.S.A, as the recipient of the 2019 Kambli, among others, passed away following a
Festival (BLF). Singapore
Japan Prize for his contributions in the field of heart attack. He was 87 and breathed his last at his
India s Ankita Raina has won the International
Biological Production, Ecolog . OBITUARIES home in Dadar. Achrekar, honoured with Padma
Tennis Federation s 25,000-dollar Women s
World s Oldest Man Masa o Nonaka Passes Shri in 2010 and Dronacharya Award in 1990, was
tournament in Singapore. In the Final, Ankita
Away instrumental in shaping the careers of Tendulkar,
defeated top seed Arantxa Rus of the Netherlands,
The world s oldest man, Masazo Nonaka of Japan, Kambli and over a dozen other top cricket stars.
6-3, 6-2. She toppled four seeded players in the
has died at the age of 113. Nonaka, who was
tournament to win her first title of the season and
officially recognized as the oldest male in the world
eighth overall.
by Guinness World Records in April 2018, passed
First Indian Athlete To Be Nominated In Laureus
awa at his home in the north of Japan s Hokkaido World Comeback of Year Award
island. Ace Indian female wrestler Vinesh Phogat has
become the first Indian athlete to receive a
Veteran Actor-Writer Kader Khan Passes Away nomination in the prestigious Laureus World
2018 ICC Awards: Virat Kohli Takes 3 Major Veteran actor-writer Kader Khan passed away in Comeback of the Year Award. She has been
Honours Canada at the age of 81. Kader Khan made his nominated alongside US Tour Championship
Virat Kohli has become the first player in history acting debut in 1973 with Rajesh Khanna s Daag winner Tiger Woods, who won his first tournament
to win the Sir Garfield Sobers Trophy for ICC and has featured in over 300 films. He wrote in five years.
Cricketer of the Year, the ICC Men s Test Pla er of dialogues for over 250 movies. As a screenwriter, 1st Away Team To Win Bilateral Series in All
the Year and the ICC ODI Player of the Year awards Pulitzer Prize-Winning Writer Mary Oliver Khan frequently collaborated with Manmohan Formats in Australia
following an extraordinary 2018. The International Passes Away India defeated Australia in the third ODI today to
Desai and Prakash Mehra.
Cricket Council also announced the men s Test and Mary Oliver, the Pulitzer Prize-winning poet Raghbir Bhola Passes Away clinch the three-match series 2-1. With this, India
ODI teams of the year with Virat Kohli named passed away. She was 83. The cause of death was Former Indian hockey player Raghbir Singh became the first visiting side to win a bilateral
captain of both sides. lymphoma. Oliver author of more than 15 poetry Bhola, a two-time Olympic medallist, has passed series (two or more matches) in all three formats in
Yu Wensheng Wins Franco-German Human and essay collections wrote brief, direct pieces away aged 92. In 2000, he was honored with the Australia.
Rights Award that sang of her worship of the outdoors and Arjuna Award for his achievements in hockey. He Chennai s Gukesh Crowned India s Youngest
A detained Chinese rights lawyer Yu Wensheng disdain for greed, despoilment and other human served in diverse roles such as the member of the Grandmaster
has won a Franco-German human rights award for crimes. Tamil Nadu lad D Gukesh became the 2nd
IHF selection committee, FIH international umpire,
his exceptional contribution to the protection and Malayalam Director Lenin Passes Away manager of the Indian hockey team, TV youngest Grand Master in the world and the
promotion of human rights. Yu Wensheng, who is Malayalam director and screenwriter Lenin youngest Indian to achieve the feat at 12 years, 7
commentator and government observer at the
in pretrial detention on subversion charges, has Rajendran has passed away at 67. He was a five- Olympic Games. months and 17 days, erasing the mark set by his
been held incommunicado by authorities in time recipient of the Kerala State Film Awards and state-mate R Praggnanandhaa in June 2018.
Jiangsu s Xu hou cit for more than a ear. also received the International Federation of Film Ukraine s Serge Karjakin remains the youngest-
Critics Award for the best Malayalam film for his ever GM, having achieved the feat at the age of 12
film Makaramanju . years and 7 months in 2002.

©LegalEdge Tutorials Page 11 of 60 ©LegalEdge Tutorials Page 12 of 60


Replication or other unauthorized use of this material is prohibited by the copyright laws of India Replication or other unauthorized use of this material is prohibited by the copyright laws of India
Current affairs Timeline Current affairs Timeline
Gurgaon s 8-year-old Kartik Singh won the under- wicket of opener Martin Guptill. The Indian record Islands. The airbase will be used to handle
8 category Junior world title at the Kids Golf was previously held by Irfan Pathan, who got there operations of helicopters and small aircraft like the
World Championship organised by US kids Golf in his 59th match, against Pakistan in Abu Dhabi in Dornier. The new airbase in the northernmost part
in Malaysia to become the youngest Indian to win 2006. The world record is currently held by of the Andaman Nicobar Islands will be used for
the title. Kartik becomes youngest Indian to win Afghanistan leg-spinner Rashid Khan, who needed both defence and civil aircrafts.
junior world titlei. Kartik is a member of the junior only 44 games to reach the mark. Shami is joint-
programme at the DLF Golf Academy. sixth with New Zealand fast bowler Trent Boult.
Mary Kom Attains No. 1 Position in AIBA SCIENCE AND TECHNOLOGY
Shakil Ahmed Sets World Record In Indoor
Rankings IIT-H Launches Full-Fledged B.Tech In Artificial
Rowing
Asian Cup gold medallist Shakil Ahmed has set M C Mary Kom has attained number one position DEFENCE AND SPACE Intelligence
in the International Boxing Association s (AIBA) India s 2nd Defence Innovation Hub to come up Indian Institute of Technology-Hyderabad (IIT-H)
up a world record in indoor rowing by completing
latest world rankings. She became the most Nashik in Maharashtra will be the site of the has announced the launch of a full-fledged
1 lakh meters in above-40 years category in Kolkata.
successful boxer in world championships histor countr s second defence innovation hub after bachelor s programme in Artificial Intelligence (AI)
Shakil began his record-setting indoor rowing feat
when she claimed the 48kg category top honors in Coimbatore in Tamil Nadu. Union Minister of State technology from the new academic session, a first
in Salt Lake stadium complex and achieved the
Delhi in November 2018. for Defence Subhash Bhamre stated that the for the country and only the third globally.
milestone of covering 1 lakh meters of simulated
Mohamed Salah Named African Footballer Of defence innovation hub at Nashik will help local Besides IIT, Carnegie Mellon University and
distance in 10 hours.
The Year industries and entrepreneurs in the countr s Massachusetts Institute of Technology (MIT), both
Khelo India Youth Games Concludes:
Liverpool s Mohamed Salah has been named as the defence sector. of which are in the US, offer full-fledged B.Tech
Maharashtra Tops Medal Tally
2018 Confederation of African Football Player of Japan Launches Satellite For World s 1st Artificial programmes in AI.
Maharashtra topped the medal tally with 37 golds
the Year for the second consecutive year. Egypt Meteor Shower
at the Khelo India Youth Games in Pune. It had 111
medals in its kitty with 37 gold, 31 silver and 43 forward Salah, 26, beat Liverpool team-mate Sadio Japan Aerospace Exploration Agency has launched Legal Affairs
Mane of Senegal and Arsenal and Gabon striker Tokyo-based startup Star-ALE s mini-satellite 1. Additional Solicitor General- Madhavi Divan
bronze. Delhi was at second position with 35 gold
Pierre-Emerick Aubameyang to the title. aimed to deliver the world s first artificial meteor Madhavi Divan is appointed as the Additional
and Haryana too maintains its third position with
Jasprit Bumrah Becomes Highest International shower. The satellite carries 400 tiny balls, will be Solicitor General of India. With this she becomes
29 gold. 10-year old Abhinav Shaw became the
Wicket-Taker in 2018 enough for 20-30 events that will glow as they rush the third female ASG after Indira Jaising being the
youngest gold medallist (shooting).
The 25-year-old fast bowler Jasprit Bumrah became down the atmosphere on being released. ALE plans first woman and Pinky Anand being the second. In
Bengaluru Raptors Won 1st Ever PBL Title
the highest international wicket-taker across to deliver its first show in Hiroshima in early 2020. the Indian Government, the post of the Additional
Bengaluru Raptors have won their first ever
Premier Badminton League, PBL title. In the title formats in 2018. He took a total of 78 wickets in the Solicitor General is ranked third after the Attorney
clash played at Bengaluru, Bengaluru edged out calendar year. He led the Indian cricket team to a General and the Solicitor General of India. Senior
Mumbai Rockets, with a 4-3 win. Star player historic victory in the Boxing Day Test at the Advocate Sanjay Jain and KM Nataraj also have
Kidambi Srikanth, Vu Ji Trang and the Men s Melbourne Cricket Ground. Bumrah surpassed been appointed as ASG by the government.
Doubles pair of Mohammad Ahsan and Hendra South Africa s Kagiso Rabada s tall of 77 wickets 2. Executive Chairman of NALSA- Justice AK Sikri
Satiawan won their matches for Bengaluru guiding to reach the milestone. While on the third position Replacing Justice Madan Lokur, Justice AK Sikri is
them to victory. is Bumrah s countr man Kuldeep Yadav who took nominated as the new Executive Chairman of the
76 scalps. India-Myanmar Joint Training Exercise Begins In National Legal Services Authority, by the President
Hopman Cup: Roger Federer & Belinda Bencic
Chandimandir Ram Nath Kovind. To implement Article 39A of the
Retain Title For Switzerland
The second edition of the India-Myanmar bilateral Indian Constitution, NALSA was formed under the
Swit erland s Roger Federer and Belinda Bencic
army exercise, IMBEX 2018-19, commenced at National Legal Services Authority Act of 1987 in
secured victory over German s Angelique Kerber
Chandimandir Military Station, that houses the order to provide free legal aid to the weaker
and Alexander Zverev to win the Hopman Cup for
headquarters of the Western Command, section.
the second year in a row. In a rerun of 2018 final,
Chandigarh. 3. CIC Chief Sudhir Bhargava
Federer beat Zverev to give the Swiss an early 1-0
Scientists have discovered a new planet twice the Sudhir Bhargava is appointed as the new Chief
lead. Wimbledon champion Kerber then overcame
Rishabh Pant Named ICC Emerging Player Of size of Earth Information Commissioner of the Central
Bencic to level the tie. However, the Swiss claimed
The Year Astronomers have discovered a planet twice the Information Commission.
a tight win in the mixed doubles to claim their
Rishabh Pant was named the International Cricket size of Earth, and it s within a one that could allow 4. Chief Justice of Various High Courts
fourth Hopman Cup title.
Council s (ICC) Emerging Pla er Of The Year liquid water to exist on its surface. The finding Acting CJ of Karnataka HC Justice L
following an impressive show in India s first-ever comes from data from NASA s Kepler space Narayana. He will replace Justice Dinesh
Test series win in Australia. The 21-year-old telescope, which ran out of fuel in October 2018. Maheshwari who has been elevated to the
wicketkeeper-batsman scored 350 runs in the four- K2-288Bb, as the new planet is called, is located Supreme Court.
match Test series that India won 2-1 to script within its star s habitable one, which is wh liquid
Acting CJ of Calcutta HC Justice Biswanath
history in 71 years and 11 attempts. water is a possibility.
Somadder. He will replace Justice Debasish Kar
Fastest Indian Bowler To Reach 100 ODI Wickets Navy Chief Admiral Sunil Lanba Commissioned
Gupta.
Mohammed Shami became the fastest Indian New Naval Air Base
5. In the Winter Session of the Parliament, the Lok
Kartik becomes youngest Indian to win junior bowler to claim 100 ODI wickets, during the first Indian Navy Chief Admiral Sunil Lanba
Sabha has passed various bills, as mentioned
world title one-dayer against New Zealand at Napier. Shami commissioned the new Naval Air Base INS below:
reached the milestone in his 56th match, with the Kohassa at Diglipur in Andaman and Nicobar

©LegalEdge Tutorials Page 13 of 60 ©LegalEdge Tutorials Page 14 of 60


Replication or other unauthorized use of this material is prohibited by the copyright laws of India Replication or other unauthorized use of this material is prohibited by the copyright laws of India
Current affairs Timeline LE Explains
The Muslim Women (Protection of Rights on purpose of this bill is to provide protection to
GAGANYAAN MISSION
Marriage) Bill - more popularly known as the the students interests who studied from an of
Triple Talaq Bill, will replace the ordinance the unrecognized institutions.
7. 103rd Constitutional Amendment, 2019 On 28 th December 2018, Union Cabinet approved a budget of Rs. 10,000 crore for India s first manned space flight mission
passed in this regard. The main aim of the Bill
The 103rd Constitutional Amendment has been which is named as Gaganyaan Mission. Indian Space Agency ISRO (Indian Space Research Organisation) is planning for a
is to protect Muslim wives by making the
passed by the Parliament and has received assent manned mission in December 2021. This will make India the 4 th nation to do so after USA, Russia and China. The mission
Triple Talaq void in all forms including
from the President which makes it a binding law in will enable ISRO to achieve higher levels of reliability in launch and satellite technology. It will help in providing
electronic and written.
the country. It was the 124th Constitutional employment to 15,000 people and out of them, 861 will be from ISRO.
The Consumer Protection Bill, 2018 this bill
Amendment Bill whose primary objective was to This ambitious project was first announced by Prime Minister Narendra Modi on 15 th August 2018 as a part of the
aims to replace the Consumer Protection Act of
provide 10 % reservation to the economically Independence Day speech to send an Indian in space by 2022.
1986 and provide better complaint Redressal
backward classes in the country. The bill has It is proposed that a Made in India spacecraft will be sent to space by 2022 which will carry three Indian astronauts who
mechanism. The bill also proposes to establish
basically amended Article 15 and 16 of the Indian will spend 7 days in the space conducting experiments on microgravity.
a committee named CCPA or the Central
Constitution which are a part of the Fundamental India has inked agreements with France and Russia for assistance in this ambitious project.
Consumer Protection Authority which will
Rights. For the success of the project, ISRO has formed a new Human Spaceflight Centre (HSC) to achieve human-rating of the
look into the enforcement of the Consumers
8. Justice Merriam Webster s Word of the ear systems. Human rating is the certification that the system is capable of transporting the humans safely into the space and
rights.
2018. has sufficient technology to efficiently protect the crew in the spacecraft in case of any failure. The Human Spaceflight
The Transgender Persons (Protection of Rights)
Merriam Webster is a US based publishing Centre will be headed by senior scientist Unnikrishnan Nair as a director and R Hutton will act as the project director for
Bill, 2016 the main objective of this bill is to
company, found by George and Charles Merriam, Gaganyaan.
provide a separate identity to the transgender
famous for its Dictionaries and references. It has It is announced that GSLV Mark III, the three-stage heavy-lift launch vehicle will be used to launch the Gaganyaan as it has
persons. It also aims at empowering them and
declared that the word of the year for 2018 would the necessary payload capability to launch a 3-member crew module in low earth orbit. The spacecraft will be placed in a
protecting them against discrimination.
be Justice . low earth orbit of 300-400 kms. For the purpose of the mission, three flights will be sent into the orbit of which two will be
The Companies (Amendment) Bill, 2018 this
9. Vijay Mallya Fugitive Economic offender unmanned flights proposed to be sent in December 2020 and July 2021 and one human spaceflight in December 2021 or
bill will replace the ordinance passed in Nov 18
Mumbai Court declared Mallya as the Fugitive early 2022. ISRO is planning to build three sets of rockets, crew and service module for the project and all of them have to
and will amend the Companies Act of 2013.
Economic offender. He is the first Indian to be be human-rated.
The Surrogacy (Regulation) Bill, 2016 the
main objective of this statute is to allow declared so.
10. China makes law for compatibility of Islam Benefits
altruistic surrogac and put a ban on the
Recently, to make Islam compatible with Socialism, Gaganyaan Mission will establish a broader framework for collaboration between ISRO, academia, industry,
commercial surrogac . The bill will also
the People s Republic of China has passed a law. national agencies and other scientific organizations.
protect the rights of the surrogate mother and
This regulation will be done b Sinici ing Islam This will allow pooling in of diverse technological and industrial capabilities and enable broader participation in
the child born out of the surrogacy.
as per Socialism. research opportunities and technology development benefiting large number of students and researchers.
Personal Laws (Amendment) Bill, 2018 this
11. For Strikes and Hartals, notice is compulsory It is expected to generate employment and train human resources in advanced technologies.
bill will remove Leprosy as a ground for
Kerala High Court It will inspire large number of young students to take up science and technology careers for national development.
divorce in all the five personal laws in India.
As per the holding of the court, the political parties
These laws include Special Marriage Act,
which intend to call for strikes or hartals, have been Strategy and Targets
Divorce Act (Christians), Hindu Adoption and
given a 7 day notice period to provide information National agencies, laboratories and Academia will participate in crew training, human life science technology
Maintenance Act, Hindu Marriage Act and the
about the upcoming strike to the public. development initiatives as well as design reviews.
Dissolution of Muslim Marriages Act.
12. Separate High Court for Telangana First human space flight demonstration is targeted to be completed within 40 months from the date of sanction.
The Citizenship (Amendment) Bill, 2019 this
From 1st January 2019, Telangana will have a Prior to this, two unmanned flights in full complement will be carried out to gain confidence on the technology and
bill aims at amending the Citizenship Act of
separate High Court. This will be the 25 th High mission management aspects.
1955. This bill will grant Indian Citizenship to
Court in India. With this Andhra Pradesh High
all the religious minorities from Pakistan,
Court will not govern Telangana. Telangana HC Impact
Bangladesh and Afghanistan.
will function from Hyderabad and the Andhra The mission is expected to spur research and development within the country in niche science and technology
New Delhi International Arbitration Center
Pradesh HC will function from Amaravati. Justice domains.
Bill, 2018 the main aim of this bill is to
TBN Radhakrishnan will be the Chief Justice of the Huge potential for technology spinoffs in areas such as medicine, agriculture, industrial safety, pollution, waste
establish the International Arbitration at New
Telangana HC. management, water and food resource management etc.
Delhi. This will replace the 1995 established
13. Women exempted from paying court fees- Odisha Human spaceflight mission will provide a unique micro-gravity platform in space for conducting experiments and
International Center for Alternative Dispute
HC test bed for future technologies.
Resolution.
Irrespective of the nationality and status, all the The mission is expected to give impetus to economic activities within the country in terms of employment
6. Parliament has passed the following bills in its
women are exempted from paying the court fees in generation, human resource development and enhanced industrial capabilities.
Winter session:
the courts of Odisha. Human Spaceflight capability will enable India to participate as a collaborating partner in future global space
The Right of Children to Free and Compulsory
14. The name of Allahabad HC will not change exploration initiatives with long term national benefits.
Education (Amendment) Bill, 2018 the bill
As a response to a RTI application, Government
removes the provision of no- detention from
clarified that Allahabad HC s name will not change
the statute.
with the change in the name of the city to Prayagraj.
The National Council for Teachers Education
(Amendment) Bill of 2018 the primary
©LegalEdge Tutorials Page 16 of 60
©LegalEdge Tutorials Page 15 of 60 Replication or other unauthorized use of this material is prohibited by the copyright laws of India
Replication or other unauthorized use of this material is prohibited by the copyright laws of India
LE Explains LE Explains
PRESIDENT S RULE IN JAMMU & KASHMIR 103RD CONSTITUTIONAL AMENDMENT ACT, 2018
Jammu & Kashmir has been in news recently for imposition of President s Rule in the state. On 20th December 2018, J&K
came under the President rule after it has completed 6 months of Governor s rule. This implies that all the polic and other 103 rd Constitutional Amendment could be said to be one of and in a way, it may be said to be class-based reservation.
governmental decisions concerning the state of J&K will be taken by the Union Cabinet. the most controversial amendments to have taken place in The step can be a political pretence in the garb of public
The Governor s rule which was imposed before the President s rule was due to the political crisis caused b the withdrawal the recent past in the Indian Constitutional Law regime. policy, as there were 4 years for the current government to
of support by BJP and its 25 members to the Mehbooba Mufti led coalition government reducing it to a minority There was a time in the Indian Constitutional History during take such a step but it was done right before the elections so
government. the tenure of Mrs. Indira Gandhi as the PM, when there used as to secure and enlarge its vote bank and include
Governor s Rule in Jammu & Kashmir to be a regular tussle between the organs of the Government Economically Weaker General Category people in it.
In India, the failure of government in a state leads directly to the President s Rule in that respective state. But, in case of and a lot of amendments in the Constitution took place, The Amendment inserted Article 15 (6) which provides for
J&K the process is slightl more nuanced as in J&K Governor s rule is imposed for 6 months before the President s rule. which caused the emergence of doctrine of basic structure reservations to economically weaker sections for admission
The Constitution of India grants a special status to the state of Jammu & Kashmir unlike other states. J&K has its own and led to the development of the concept of judicial review to educational institutions including private educational
separate constitution and Penal Code as well. of the legislation. After this era, it seems that the time for our institutions, whether aided or unaided by the State, other
Section 92 of the Jammu and Kashmir Constitution provides for the Governor s rule in case of failure of the state judiciary has come again to review a Constitutional than the minority educational institutions referred to in
government. The proclamation is issued by the governor of the state after President Assents to the same. Amendment proactively and this time the petition for its clause (1) of Article 30. The amendment aims to provide
review was filed in the Apex Court itself even before it got reservation to those who do not fall in 15 (5) and 15(4)
Under the Governor s rule the states assembl remains either suspended or dissolved. Thereafter, if the governor s rule
the Presidential Assent. 1 Thereafter, as soon as it became (effectively, SCs, STs and OBCs). It further added Article 16
continues for six months and there is no government formed within the 6 month period, the President s rule is imposed
operational and was inserted into the text of Constitution, (6) to provide reservations to people from economically
under Article 356 of the Constitution ending the Governor s rule.
the advocates and activists filed three more petitions. 2 As far weaker sections in government posts. Furthermore,
First Governor s rule in Jammu & Kashmir was imposed in March 1977, when the congress withdrew the support from the as the status of amendment is concerned, the Supreme Court an explanation has been added, which states the criteria to be
late Sheik Abdullah led National Conference (NC) government. refused to order a stay on the operation of the amendment; listed as "economically weaker". The criteria, though has not
Other notable differences in Jammu & Kashmir from the other states in India, the head of the state was called Sadr-e-Riyasat however, it has assured the petitioners of examining the been defined in any of the documents in black and white and
instead of the Governor and the head of government in the state was called the Prime Minister instead of Chief Minister in amendment in the light of the constitutional principles and the closest the Act has defined it is in the explanation;
other states. precedents in this regard. however, according to the ANI quoting Thawarchand
President Rule in Other States Gehlot (Minister for Social Justice and Empowerment), the
Article 356 of the Constitution provides for the President s rule, it means direct imposition of the rule of Central government What is 103 rd Constitutional Amendment? criteria which has been circulated is as follows-
in the state in which the rule is imposed. It is done when the state government becomes incapable of functioning according The 124 th Constitutional Amendment Bill, which
to the provisions of the Indian Constitution. subsequently became the 103 rd Constitutional Amendment Family income should be less than Rs 8 lakh per
While proper functioning of the state government according to the constitutional provisions, the elected council of ministers Act, seeks to provide a significant 10% quota to the annum
are responsible for the functions of state s legislative assembl led b the Chief Minister being the chief executive of the economically weaker section in the General Category in the Farm land should be less than that of 5 acres
state. The governor acts merel as a constitutional head in that case. When the President s rule is imposed the Council of Government Jobs and Educational Institution. This may be Residential house should be less than 1,000 sq ft
Ministers is dissolved and the office of Chief Minister stands vacated. said to be a step towards its own political agenda by the Residential plot should be below 100 yards in a
Reasons for imposition of President s rule. current government right before the General Elections in notified municipality
1. When the state legislature fails to elect a leader or Chief Minister due to lack of majority. May 2019 but whatsoever the intent may have been, the step
In case of a non-notified municipality area, the
2. In case when the government formed is a Coalition government and 2 or more parties came together to form the has been getting a mixed response.
residential plot should be below 200 yards
government the withdrawal of any of any of the parties may lead to a breakdown in the government as the
The Constitutional (103rd Amendment) Act got the assent of
remaining party will be in minority. The same happened in Jammu & Kashmir when the BJP withdrew its support. These criteria is largely criticised as it is alleged that for a
President of India on 13th January, 2018. The bill was passed
3. When the ruling party in the state fails to prove its majority in the state legislative assembly it will lead to the fall country like India, this is a very high threshold and
in Lok Sabha by 323 members voting in favor and 3 members
of elected government in the state thereb attracting president s rule. practically everyone shall be included in this category of
against the bill. Rajya Sabha subsequently passed it with 165
4. When due to some unavoidable reasons the elections of any particular state are postponed the president rule shall Economically weaker even though he/she is not-so-poor in
members in the favor and only 7 members against the bill.
be imposed for the period till a new leader is elected. reality.
This shows that the majority in both the houses agreed on
Tenure of President s Rule. taking the step. However, the petitions filed in the Apex
1. President s rule is imposed for a period of 6 month after it has been approved by both the houses of the parliament. The Apex Court while not putting a stay on the operation of
Court challenge the step as they contend that it abrogates the
2. The president s rule can be extended for 3 ears with an approval of both the houses of the parliament which has this amendment has issued notice to the Union Government
basic structure of the Constitution in exceeding the 50%
to file their reply against the petitions filed in the Court
to be done every 6 month. ceiling on reservation as imposed in the 1993 Indira Sawhney
challenging the amendment and it has also bought time for
3. President rule can be revoked anytime by the President without the consent of the Parliament. case.
itself to examine the validity of the amendment. It would be
4. To put restrictions on the power of the Parliament to extend the rule of President in any particular state 44 th
interesting to see if the apex court examines the amendment
Amendment Act 1978 introduced a provision. According to this provision, President rule can be extended for a The Supreme Court in Indira Sawhney Case had put a 50%
in the light of the constitutional principles and its own
year after every Six months only when: cap on caste-based reservations and ruled, No provision of
precedents in this regard or would just leave it as it is by
a. There is already a National Emergency in force throughout the Country, or the whole of a state or in any reservation or preference can be so vigorously pursued as to
taking the defence of utilitarianism and separation of power
particular part of it. destro the ver concept of equalit . The government can
principles. Whatever the result be, one thing is for sure that
b. The Election Commission has certified that due to some valid and unavoidable reasons elections cannot be make the defence to this step by contending that the
the practical implication of this reservation would not be too
conduct in that particular state. reservation is based on the economic condition and not caste,
significant as the scenario would not deviate much, i.e., huge
however, such a condition is made for general category only
amount of people running to get a few number of jobs.

©LegalEdge Tutorials Page 17 of 60 ©LegalEdge Tutorials Page 18 of 60


Replication or other unauthorized use of this material is prohibited by the copyright laws of India Replication or other unauthorized use of this material is prohibited by the copyright laws of India
LE Explains LE Explains
# Stare decisis is a legal doctrine that obligates courts to follow historical cases when making a ruling on a similar
current or future case. Stare decisis ensures that cases with identical facts be approached in the same way, unless
0 LEGAL ONE LINERS
1. Be ond 200 miles from the coast, is known as High Sea . overruled by the higher court.
2. Joint Parliamentary Committee is chaired by a member of the Opposition Party. 7. Status Quo- the existing state of affairs
3. 21 st Amendment recognised Sindhi as one of the languages under the 8th Schedule of the Constitution of India, # The Latin term status quo refers to the existing state of circumstances. In the legal system, a judge has the
1950. authority to issue a status quo order to prevent anyone from taking any action until the matter can be heard and
4. The Supreme Court was created by the Constitution of India, 1950. resolved by the court. A status quo order preserves the existing situation, so that no part s position can be
5. A retired judge of a High Court cannot practice in the High Court from where he has retired. compromised or prejudiced until the matter has been resolved.
6. Doctrine of Estoppel means restriction to make statement contrary to the earlier statement/admission. 8. Corpus Juris- body of law
7. National Flag was approved and thereby accepted by the Constituent Assembly on 22nd July, 1947. # It was originally used by the Romans for several of their collections of all the laws in a certain field. The law in
8. Finance commission constituted under Article 280 of the Constitution has one Chairman and four other Members. general, especially when compiled, codified, and published in a single text or in a series consisting of a collection of
9. Guidelines pertaining to prevent sexual harassment of working women was laid down by the Supreme Court in the individual laws become corpus juris.
case of Vishaka v. State of Rajasthan. 9. Escheat- a common law doctrine where in the property of a person dying without heir is transferred to the state.
10. A Member of Parliament of India or Member of Legislative Assembly of a State cannot be arrested in a civil action # If A dies leaving no living heir behind him, his property will directly go to state. It serves to ensure that property
within a period of 40 days before or after the session. is not left in "limbo" without recognized ownership. It originally applied to a number of situations where a legal
11. The maxim sic tari tuo alienum non lecdus means enjoy your own property in such a manner as not to injure the right interest in land was destroyed by operation of law, so that the ownership of the land reverted to the immediately
of another. superior feudal lord.
12. The Committee on the Rules of Procedure formed by the Constituent Assembly was headed by Dr. Rajendra 10. Pendente Lite- During the pendency of the litigation
Prasad. # Pendente lite is a Latin term meaning "awaiting the litigation" or "pending the litigation" which applies to court
13. During the British Rule, the Rowlatt Act provided that even without trial a person can be arrested. orders which are in effect while a matter (such as a divorce) is pending. In divorce a pendente lite order is often
14. The Apex Court in the case of Lily Thomas v. Union of India, (2013) 7 SCC 653, held that on conviction of offences, the used to provide for the support of the lower income spouse while the legal process moves ahead.
MPs and MLAs will be immediately disqualified from holding membership without being given three months.
15. Transfer of some existing movable/immovable property without consideration is known as Gift.
16. Facts which are inconsistent with any fact in issue are relevant, is based on the principle of plea of alibi. KEY DIFFERENCES BETWEEN IMPORTANT CONCEPTS
17. The provision regarding emergency was adopted in the Constitution of India from the Constitution of Germany.
1. Partnership and Limited Liability Partnership (g) In an LLP, no partner is liable for the
18. Recently, the Registrar of Geographical Indications Registry has conferred GI tag on Banganapalle Mango of
(LLP) independent or unauthorized actions of other
Andhra Pradesh.
(a) Partnership is the relation between persons partners; whilst on the other hand; a
19. The 99th Amendment of the Constitution provides for National Judicial Appointment Commission.
who have agreed to share the profits of a partnership is based on agency relationship
20. Raja Challiah Committee is related to tax reforms.
business carried on by all or any of them where every partner is responsible for the acts
acting for all, while LLP is an alternate form of of other partners.
Maxims and Foreign Phrases business which combines the attributes of
1. Causa Causans- the immediate cause of something both partnership and a company. 2. Wagering Agreements and Contingent Contracts:
# In Causation in law of torts, court always looks for the immediate cause of damage and not the remote cause to (b) An LLP is governed by the Limited Liability (a) Section 30 of the Indian Contract Act defines
decide the liability. Partnership Act, 2008 and a partnership is wagering agreements, while Section 31
2. Ex Gratia- payment made without any legal obligation like out of pity governed by the Partnership Act, 1932. defines contingent contracts.
# When something has been done ex gratia, it has been done voluntarily, out of kindness or grace. Compensation (c) An LLP shall be registered under the LLP Act, (b) Contingent contract is a valid contract, while
payments are often made ex gratia if a government or organization is prepared to compensate victims of an event 2008 by complying with the regulations wagering agreements are void in nature.
such as an accident or similar but not to admit liability to pay compensation or for causing the event. prescribed therein whereas a partnership may (c) A wagering agreement is a game of chance;
3. Non est factum- not his deed or may not be registered. the parties have no other interest in the subject
# It is a defence in contract law that allows a signing party to escape performance of an agreement which is (d) Liability of partners in a partnership firm is matter of the agreement except winning or
fundamentally different from what he or she intended to execute or sign. A claim of non est factum means that the unlimited (explained above) while in LLP, the losing of the amount wagered. However, a
signature on the contract was signed by mistake, without knowledge of its meaning. liability of partners is limited to their contingent contract is not a game of chance but
4. Persona non grata- an unacceptable person contribution as agreed upon in the agreement. it is a contract to do or not to do something if
# In diplomacy, a persona non grata is a foreign person whose entering or remaining in a particular country is (e) A minimum of two partners will be required some event collateral to such contract does not
prohibited by that country's government. Being so named is the most serious form of censure which a country can for formation of an LLP. There will not be any happen.
apply to foreign diplomats, who are otherwise protected by diplomatic immunity from arrest and other normal limit to the maximum number of partners. The (d) Under a wagering agreement, a future
kinds of prosecution. maximum number of people in a partnership uncertain event is the key element of the
5. Res Judicata- a matter already decided firm is 10 in case of a banking business and 20 agreement. In a contingent contract, future
# Once adjudicated, the same matter cannot be brought to the court again. The doctrine of res judicata is a method in case of all other types of businesses. uncertain event is collateral to the main
of preventing injustice to the parties of a case supposedly finished, but perhaps also or mostly a way of avoiding (f) The LLP is a body corporate with a legal entity purpose of the contract.
unnecessary waste of resources in the court system. Res judicata does not merely prevent future judgments from separate from its partners with perpetual (e) In a wagering agreement, reciprocal promises
contradicting earlier ones, but also prevents litigants from multiplying judgments, and confusion. succession, while a partnership s existence is are definite. But in a contingent contract, there
6. Stare decisis- the doctrine that the decisions of the superior courts to be followed in subsequent cases of similar dependent on the individuals that comprise it. may or may not be reciprocal promises.
nature.

©LegalEdge Tutorials Page 19 of 60 ©LegalEdge Tutorials Page 20 of 60


Replication or other unauthorized use of this material is prohibited by the copyright laws of India Replication or other unauthorized use of this material is prohibited by the copyright laws of India
LE Prep Assist LE Prep Assist

Mentor View- English Language Mentor View- Mathamatics


How to memorize and revise words.
One of the most underrated and least attended to aspects of learning scholastic material is revision. Revision is more important Let s talk how we can improve our scores in quantitative aptitude (mathamatics) section of the CLAT Exam. Most of the
than memorization because it reminds your brain what it needs to store for the long term thereby sealing it in there. Your brain students here who choose law as their career are in general either afraid of math or it does not entice them as much as
will store only what it thinks is important and you get to decide what is important by doing that thing repeatedly and at regular legal or English does. But ou don t have an other option. You cannot ignore this section if ou are aiming for a decent
intervals. score in CLAT or AILET. Actually the marks of this section are bonus marks because the best thing about math is that
How to revise? First recall all the words that you can. You can use flash cards for this purpose or do it classic style by writing the there is less chance of negative marking as you do not answer these questions based on gut feeling but by solving them.
words down first and then writing the meanings of the words that you remember. If ou aren t sure about the meaning of a Here are a few tips which can help you score good marks in this section and if you follow them diligently you may even
particular word but you think that you know it, write it down too and mark it with a star. After you are done, check how many score full marks.
you got right and how many were wrong. Only after you have done this, move on to the next step.
Words that you tested yourself on can be classified into four major types.
Basic concept: To cover the basic concept you can start with R.S Aggarwal and my suggestion would be that you
i.) The ones you got right and were sure about
complete it twice before February ends (hopefully you have already completed it at least once).
ii.) The ones ou weren t reall sure about but got them either right or wrong.
iii.) The ones you got wrong but were really sure about
Solving previous year questions: It is very important to solve previous year question papers because they give a clear
iv.) The ones that sounded completel foreign to ou or ou couldn t recall at all
The first category of words are the ones you have really committed to memory. You either knew them beforehand or they are
idea about the kind of questions that have been asked previously and of the variations in question topics. You may expect
simply the ones that your brain decided to store without much hassle. Just keep revising them once in a while. Twice weekly for certain new topics to feature as well which haven t been asked so far such as Core Algebra or Core Geometr .
the first two weeks then once weekl and then once ever month, though this is mere advisor . I d recommend that ou revise
them as soon as you start feeling that it has been a while since you last read them. Short tricks: Now when I talk about short tricks that does not mean to know a trick for each type of questions but to
The second categor of words are the ones that ou recogni ed but weren t reall sure what the meant. This means that ou change your strategy to a more efficient and less time consuming one. Like if we solve a question of profit and loss or SICI
need to work now solely on associating that word with the meaning. It can be done in a variety of ways, some of which I have (Simple interest and compound interest) it may be lengthier when you try to solve it by basic method but when you solve
discussed below. it with ratio or mixture and allegation method it can be solved within a few seconds.
The third category of words are usually the ones you mistook for some other word. One reason for this could be that it has been
a while since you last revised them so your memory is now fuzzy; in which case all you need is a few revisions. This set of Practice of advance level questions: This is for those students who are targeting a 16+ score in CLAT or an 8+ score in
words often contains words often confused with each other. So keep this list handy and use the above techniques to distinguish AILET. You should practice qualit questions that ou can get in RSMS (Rankers stud material) and in Arun Sharma s
and memorize them. They will also be useful for confusable type questions. book on Quantitative Aptitude.
The fourth set of words are the ones that ou reall need to work on. If ou haven t read them thoroughl , do that. If ou have
and they still feature in the fourth set, you probably need to change the strategy that you used to memorise them. Refer to the It is advisable to read the following chapters in the below prescribed order:
following list of plausible strategies for the same: 1. Simplification/Percentage/ profit and loss/ discount/ SI CI/Average /Time speed and distance
1) Rote learning: Classic mugging up. Works 10X better if you mug them by writing 2. Time and work/ Pipe and Cistern/ ratio/ /partnership/ Number system/ mixture and allegation
2) Root word association: This is especially well suited for the fourth category of words as it has relative lesser number of 3. Geometry /mensuration /coordinate geometry /Height and distance/Stock and shares
words. Look up the etymology of the hard to memorize word on google and the result should be interesting enough in most
4. Permutation and combination/ probability/clock/ Data interpretation
cases to easily remember the word and its meaning.
3) Daily usage: Simply use the word in day to day language, Make sentences with them and/or use them to describe things
Keep the following two concepts in your mind when you start your day
around you that you otherwise ignore or take for granted. If that is unlikely, you can even use them instead of the words
Concept 1 Your graph of study should go like the graph of that means your study time should go on increasing
that you use daily in your mother tongue.
4) Word-imagery association: The more ridiculous the imagery that the word sprouts in your brain is, the sooner you will
learn the word. I will give an example of this technique as well as the daily usage technique at the end of this write-up.
5) Sensory inclusion: The more number of senses you involve in the learning experience (in reality or in imagination), the
easier it is to remember things. In case of words you can use the sense of sight to conjuring up an image in your imagination
to associate with a word, sense of feeling, taste and smell if describing something tangible. Sense of touch and sound by
enunciating the words clearly and feeling every movement in your mouth and coordinating it with the sound that is made.
Writing down what you have learned is an excellent way to engage sense of touch and sight simultaneously. You will get a
sense of what I am trying to explain in the example given below too.
An example of word to imagery and daily usage techniques:
I chose to learn a new word, Farrago. (And it won t come as a surprise to ou that I learnt it from a tweet who else but Shashi Concept 2 Mind bowl concept It says that our mind is like a bowl. We fill it with the information and concepts but some
Tharoor). According to vocabulary.com, a farrago is a disorganized mix of things that don't fit together. Now to commit it to of them leak out so we must refill the bowl with what has leaked out. In a nutshell you have to revise those topics which
long term memor , I imagined farras (Hindi word) or hundreds of pieces of white paper falling from above and an Italian you are not comfortable with or keep forgetting again and again.
voice screaming in Italian accent, a farraaaagoo , referring to the disorgani ed mess in the air. Another imager I used was of
m self as a child pla ing with hundreds of colored bounc balls or marbles in a tub and exclaiming in a jo ous voice, A Do keep in mind that hard work does pay off but smart work pays you better
farrago . That does wonders for memorization but now the next step is to find ways to use it in daily life, several times in a day.
So if I am passing b a stranger, whose hair is a mess, I might exclaim to m self, that was a farrago of hair! . Or ma be at the
end of a class when I look back at m board work, I might secretl think, ma be the word farrago was invented for times like Rishabh Shukla
these . As far as using it in a sentence is concerned, It was reall hard to prevent this write up from turning into a farrago of SME Mathamatics
ideas Did I succeed? Do let me know. Yuvraj Bhatia LegalEdge Bhopal
SME English Language

©LegalEdge Tutorials Page 21 of 60 ©LegalEdge Tutorials Page 22 of 60


Replication or other unauthorized use of this material is prohibited by the copyright laws of India Replication or other unauthorized use of this material is prohibited by the copyright laws of India
LE Prep Assist LE Prep Assist

Prep Test Legal Aptitude (d) None of the above.


Time: 30 min. No. of Questions: 50 Principles: (Q 13 and Q 14): Intentional torts are intentional acts that are reasonably foreseeable to cause harm to an individual and
that do so.
1. The most essential feature of the Parliamentary form of government is the: Explanation: In most cases, transferred intent, which occurs when the defendant intends to injure an individual but actually ends up
(a) Independent Judiciary injuring another individual, will satisfy the intent requirement.
(b) Sovereignty of the Parliament Facts: Tinu, an infamous assassin was given Rs.30,000 to beat up Samrat, a businessman. Tinu was waiting outside a
(c) Accountability of the Executive to the Legislature restaurant where Samrat had gone for dinner, with a plan to ambush him. When Samrat walked out, Tinu dragged him to
(d) Written Constitution a corner and began beating him but just then, Samrat s friend Kamran saw this and ran to help Samrat. He got hit with the
rear end of the pole with which Tinu was thrashing Samrat when he intervened in the scuffle.
2. If an Indian citizen is denied a public office because of his religion, which of the following Fundamental Rights is
denied to him? 13. Which of the following is correct?
(a) Right to Freedom I. Direct intent was involved in Tinu s actions
(b) Right to Equality II. Transferred intent was involved in Tinu s actions
(c) Right to Freedom of Religion III. Samrat suffered from a reasonably foreseeable harm
(d) Right against Exploitation IV. Tinu cannot be held liable for an intentional tort
(a) I,III (b) I,II (c) I,II,III (d) II,IV
3. What is the age for contesting election for the Lok Sabha?
(a) 18 yrs (b) 28 yrs (c) 35 yrs (d) 25 yrs 14. Facts: Say, Tinu while waiting to beat up Samrat, nabbed the wrong person and started beating him up instead. Will
the victim be successful in filing a tortious claim against Tinu?
4. Which among the following is another unofficial name of Rajya Sabha? (a) Yes, because Tinu s actions involve transferred intent
(a) House of People (b) House of Marksmen (b) Yes, because Tinu s actions involve direct intent
(c) House of Elders (d) House of Dividends (c) No, because Tinu did not intend to hurt the other person
(d) It cannot be determined from the facts
5. Who administers oath to the Governor of a State?
(a) President of India (b) Chief Justice of the State High Court Principles: (Q 15 and Q 16):
(c) Advocate General of the State (d) None of the above Trespass to chattels (goods or personal property) is an intentional interference with the possession of personal property, proximately
causing injury.
6. The election to the office of the President is conducted by: There are three elements to it:
(a) The Speaker of Lok Sabha (b) The Election Commission of India 1. Lack of consent in the interference with the property
(c) The Prime Minister's Office (d) The Minister for Parliamentary Affairs 2. The interference with the property must result in actual harm
3. The interference must be intentional
7. The first Country in the world to implement Right to Information law is: 15. Facts: Sabrina was a busy legal professional. She bought an expensive clock from Switzerland and told her butler,
(a) Canada (b) Finland Edward, that it needed to be oiled from time to time, and he should take care of it. Edward took the clock to a nearby
(c) Denmark (d) Sweden shop for repair one day, and Sabrina, on finding the clock missing, told him off and said she would sue him for trespass
to chattel.
8. What kind of Emergency the Union of India cannot impose on the State of Jammu and Kashmir? (a) Sabrina may sue Edward for trespass to chattel as he did not follow instructions
(a) Financial Emergency (b) State Emergency (c) National Emergency (d) War Emergency. (b) Sabrina may sue Edward for trespass to chattel as he could have harmed the clock
(c) Sabrina may not sue Edward for trespass to chattel as there was implied consent by her
9. As per the Constitution of India, a citizen of India should not be less than__________of age to become the President of (d) Sabrina may not sue Edward for trespass to chattel as merely admonishing him would suffice
India?
(a) 30 years (b) 35 years (c) 40 years (d) 45 years. 16. Facts: Wilbur left his log book on his desk in the office. His colleague, Wilson, saw it lying there and thought he would
take it and cross-check the records, even though it belonged to Wilbur. While checking the records, Wilson accidently
10. Apart from Jammu & Kashmir, which of the following States has a special protection, notwithstanding anything spilled ink on the logbook and it was damaged to a great extent. It was the only copy Wilbur had.
contained in the Constitution, in certain matters? (a) Wilbur may sue Wilson for trespass to chattel as he intentionally interfered with his property without consent and
(a) Tripura (b) Himachal Pradesh (c) Chhattisgarh (d) Nagaland. caused harm to it
(b) Wilbur may sue Wilson for trespass to chattel but Wilson will only be responsible for taking the logbook and not
11. The National Commission to review the working of the Constitution of India was headed by: spilling ink as it was an accident
(a) Justice R.S. Sarkaria (b) Justice H.R. Khanna (c) Wilbur may not sue Wilson for trespass to chattel as the logbook was a common office document
(c) Justice Chinnappa Reddy (d) Justice M.N. Venkatachalaiah. (d) Wilbur may not sue Wilson for trespass to chattel as no actual harm resulted from his actions

12. Which among the following is provided for in the Constitution of India? Principle: The doctrine of Ex turpi causa non oritur actio is the illegality defence, stating that no right of action arises from a despicable
(a) Communal electorate cause.If a person is involved in a wrong deed at a time when the alleged negligence occurred, the defendant s liability may be considerably
(b) Separate electorate for depressed class reduced or extinguished completely.
(c) Reservation for legislative seats for SC & ST

©LegalEdge Tutorials Page 23 of 60 ©LegalEdge Tutorials Page 24 of 60


Replication or other unauthorized use of this material is prohibited by the copyright laws of India Replication or other unauthorized use of this material is prohibited by the copyright laws of India
LE Prep Assist LE Prep Assist
17. Facts: Lawrence lived next to Rupert. He was bugged by Rupert playing music all the time but never really mentioned an agreement. Explanation.- It is immaterial whether the Indian Penal Code is or is not in force in the place where the coercion is
the problem to Rupert. One da , he crawled into Rupert s house and broke his music system to get rid of the problem employed
once and for all. Soon enough, he heard Rupert s footsteps and Rupert, sensing something was wrong, called out, II. Section 309 of Indian Penal code provides that whoever attempts to commit suicide and does any act towards the commission of
Hello! Who is there? This startled Lawrence. He hid near the fireplace and accidentl burnt himself. such offence, shall be punished with simple imprisonment for a term which may extend to one year 1 or with fine, or with both.
I. The doctrine of Ex turpi causa non oritur actio will apply in the instant case 20. Facts: Vidya who lives in Srinagar with her mother Kanta, threatens Kanta that if she does not sell her house to Vidya
II. Rupert will be liable Lawrence s burn injuries for INR 50,000 or less, Vidya will commit suicide. Does Vidya's act amount to coercion?
III. Rupert s liabilit for Lawrence s injuries will be mitigated or extinguished
IV. It cannot be determined (a) No, since Vidya is threatening to commit suicide; the Indian Penal Code on the other hand provides that only attempt
to commit suicide is an offence and therefore actually committing suicide is not an offence under the Indian Penal
(a) I, III Code.
(b) II,III (b) No, since they live in Jammu and Kashmir where the Indian Penal Code is not applicable.
(c) IV (c) Yes, since Kanta will have to enter the agreement without free consent.
(d) I (d) Yes, since there was a threat of attempt to commit suicide

Principles: (Q 18 and Q 19): Principle: When two or more persons, by fighting in a public place, disturb the public peace, they are said to commit an affray.
I. When a contract has been broken, the party who suffers by such breach is entitled to receive, from the party who has broken the 21. Facts: William and Christina, husband and wife, were having a severe fight in the terrace of their house. The terrace
contract, compensation for any loss or damage caused to him thereby, which naturally arose in the usual course of things from such was only on the first floor and was clearly visible from the street. The voice from their shouting reached not only the
breach, or which the parties knew, when they made the contract, to be likely to result from the breach of it. street but most houses in the vicinity. Eventually, their fight turned violent and Christina started throwing stuff at
II. Such compensation is not to be given for any remote and indirect loss or damage sustained by reason of the breach. William who ducked and a piece of a metal lamp followed by kitchen utensils started falling on the street below.
18. Facts: Faraz entered into an agreement with Karn on 03 October whereby he would provide Karn with 50 new and very Although nobody got hurt, it was because everybody on the street was aware of and was watching the fight. The
successful DSLR cameras of specified make. However, the cameras were to be provided on 03 January of the next year, children in the nearby houses were not able to study and the old and the sick were not able to sleep because of the fight.
on which date, after the delivery of the cameras, Karn would pay Faraz INR 12,80,000. However, on 03 January, Faraz Are William and Christina committing affray?
calls Karn and tells him that he will not be able to provide the cameras now or anytime in the foreseeable future. Karn, (a) No, since terrace of their house is not a public place.
who has already taken advance orders from customers on the basis of Faraz's promise of delivering cameras, finds (b) No, since they are husband and wife and their fight is a domestic affair.
himself in a soup. He contacts another wholesale supplier of DSLR cameras, Chandan, and finds out that an order of (c) Yes, since the peace-disturbing effects of their fight can be felt in the street as well, which is a public place.
50 cameras (of the same make as he had ordered with Karn) will cost INR 13,50,000 and the cameras will be delivered (d) Yes, since nuisance is being caused in the entire neighbourhood.
in batches of 10 on Saturdays of the following five weeks. The payment will have to be made at the end of five weeks
when all of the cameras are delivered. Although Karn places the order, he is agitated and sues Faraz for breach of Principle: An inevitable accident is that which could not possibly be prevented by the exercise of care, caution and skill.
contract. What is the nature of compensation that Karn will get from Faraz? 22. Facts: Neel was driving a heavy truck carrying zomide, a chemical used in manufacturing of batteries. Zomide had the
(a) INR 70,000 property of turning gaseous on contact with open air and remained liquid only in a pressurized container such as the
(b) INR 12,80,000 one it was being transported in on the truck. In it gaseous form, zomide was lethal to human beings. Neel had
(c) INR 13,50,000 instructions to deliver the zomide at the manufacturing plant at Sopanabad the same day. However, when he reached
(d) INR {13,50,000 + 13,50,000 x 0.028 (average rate of inflation pertaining to the said period of three months)} Sopanabad, he learnt that some of the oil tanks at the manufacturing plant had burst the previous night and the plant
was surrounded in oil, including all three roads that led to the plant. He talked to one of the colleagues who had been
19. Facts: In the same set of facts as above, one of the customers Mr Salve, to whom Karn had promised delivery of camera coming out of the plant who told him that there was a lot of oil everywhere, but the least on the road at the back entry
on the same day, 03 January, punctually arrives at his shop for collection. However, Karn tells him of the unavailability to the plant. Neel decided to follow the safest route and drove all around the manufacturing plant through busy lanes
of the camera. He is very disappointed and goes back home. The camera was actually a gift for his son Ritesh who had and narrow turns reaching the rear entry road to the plant. However, as he cautiously approached the plant, the tyres
recently got a job in a company. When Mr. Salve conveys the news to Ritesh, he also feels very disappointed and decides of the truck suddenly skidded badly on the oil and the slightly tilted road and the truck toppled over beside the road.
to go to the store and talk to the owner, Karn. He takes along one of his friends, Pranay and goes to the shop. He finds The zomide container hit the ground hard and cracked, letting zomide leak. Over the next few hours, hundreds of
Karn and tells him that it is very unprofessional to take advance money and not deliver the camera on time. Karn says people in the city choked to death after inhaling zomide. Was there an inevitable accident?
that he understands and will ensure that the camera is with Ritesh by next saturday. However, in the meantime, Pranay (a) Yes, since Neel chose the road with the least amount of oil and drove cautiously.
sees Karn's helper whom Pranay has seen misbehaving with Pranay's sister on the streets for a few weeks; He grabs (b) Yes, since Neel was not aware of the road being slightly tilted.
hold of him and an altercation ensues; several glasses, phones and cameras in Karn's shop are broken. Karn in his suit (c) No, since considering the lethal nature of goods being carried and the slippery nature of oil, Neel should not have
for damages, pleads that the damage to his inventory amounted to INR 1,70,000 and was caused because of the breach tried to enter the plant.
of contract by Faraz who should therefore be made to compensate for the same. Will Faraz be made to compensate of (d) No, since due to Neel's negligence, hundreds of lives were lost.
this damage?
Principles: Employers are vicariously liable for the torts of their employees that are committed during the course of employment.
(a) Yes, since but for Faraz's breach of contract, the said damage would not have occurred. 23. Facts: X is the owner of a circus. He travels all over the country with his crew and conducts shows. For the purpose of
(b) No, since the damage is too remote and indirect. transporting his animals, he uses the services of a transport agency run by Y. On one of such occasions, when Y had
(c) Yes, since some of the cameras and phones broken were last pieces of popular series and therefore were in demand. sent one of his trucks from Delhi to Chandigarh for transporting the animals, the driver, Z, carelessly went to have tea
(d) No. since Karn should have been more polite in dealing with his customers. while the cage wasn t closed properl . He had been advised to be careful during the discharge of duties by his boss, Y,
many a times. There were two horses who fled from the cage. Consequently, X sues Y for damages.
Principles: (a) Y shall be held liable vicariously for the negligence of Z.
I. "Coercion" is the committing, or threatening to commit, any act forbidden by the Indian Penal Code, or the unlawful detaining, or (b) Y shall not be held liable as it was Z s negligence which caused the damage.
threatening to detain, any property, to the prejudice of any person whatever, with the intention of causing any person to enter into (c) Y shall not be held liable as by advising Z to be careful Y had taken requisite care.

©LegalEdge Tutorials Page 25 of 60 ©LegalEdge Tutorials Page 26 of 60


Replication or other unauthorized use of this material is prohibited by the copyright laws of India Replication or other unauthorized use of this material is prohibited by the copyright laws of India
LE Prep Assist LE Prep Assist
(d) None of the above. II. A contract entered on the basis of such active concealment is voidable at the option of the party whose consent was obtained on the
basis of such fraud.
Principle: Employers are vicariously liable for the torts of their employees that are committed during the course of employment. 28. Facts: There is a house in the city of Ramnagar and its owners are desperate to sell it. However, no one is willing to
24. Facts: X is a reputed swimming instructor with a number of sub-instructors working as employees in his academy. One buy it as everyone in the town has heard stories of it being haunted. The owners engage a real estate agent, Y, who
da , X wasn t around and his sub-instructor Y was looking over the students of the academy. He got a phone call and gets an NRI couple interested to buy the property. The couple has no idea about the house being labeled haunted.
left the swimming pool to attend it. Meanwhile, one of the kids- A, started drowning. By the time, Y reached and However, the husband, X causall asks the agent, There isn t an thing spook with the house, right? M wife gets
rescued the child, he had already fainted. He had to be rushed to the hospital and it took a couple of days for him to reall creeped out b ghost stories. Y just laughs and goes away to make a phone-call. After the agreement is
completely recover from it. His parents sued X. finali ed, a local tells X s wife about the house and wh the are getting it at such a low price. She tells X and the
(a) X cannot be held liable as the mistake was committed by his sub-instructor, Y. immediately cancel the deal. The owners sue X for breach.
(b) X shall be held liable as the tort was committed by his employee during the course of employment. (a) The contract is void-ab-initio and X cannot be held liable.
(c) X shall be held liable as he should always be around the swimming pool and take care of the students. (b) X will not be held liable as the contract is voidable at his option.
(d) X shall not be held liable as he was not present at the scene of the incident and thus, it cannot be said to be his fault. (c) X will be held liable as it wasn t the owners who concealed the facts and therefore, the should not suffer.
(d) None of the above.
Principle: It is the duty of the Municipal Committee to take proper care of buildings so that they should not prove a source of danger
to persons. Principle: If someone willingly places themselves in a position where harm might result, knowing that some degree of harm might
25. Facts: There is a clock tower in a dilapidated state in the middle of the town of Ramnagar. The residents have written result, they are not able to bring a claim against the other party in tort.
several letters to the Municipal Committee of Ramnagar complaining about the condition of the clock tower and that it 29. Facts: A is a thirteen ear old girl. She takes admission in B s karate academy. She asks B to teach her the tile breaking
could fall any time. However, their concerns fell on deaf ears. One of the reasons given by the Municipal Committee trick to which he says that she needs a couple of weeks practice before she can try that. However, A is extremely
was that they lacked the funds for the renovation, which was a true fact. One fine day, the structure collapsed, hurting curious and tries to break three tiles after the class ends and when no one is around. Instead of breaking the tiles, she
a couple of people. Mrs. X s son, Y was also injured and she filed a suit for negligence against the Municipal Committee. ends up getting a fracture. Her parents sue B.
(a) The Committee shall be held liable for negligence as they owed a duty of care to the residents of Ramnagar. (a) B shall not be liable.
(b) The Committee shall not be held liable as the residents should have been careful around the structure which was (b) B shall be liable as he kept dangerous things in his academy.
very obviously in a bad shape. (c) B shall be held liable because had he not refused to teach her the trick, she wouldn t have hurt herself.
(c) The Committee shall not be held liable as they had no ill-will towards the residents and could not renovate due to (d) B shall be held liable as he is responsible for the safety of kids in the academy.
sheer lack of funds.
(d) The Committee shall be held liable because they did not respond to the letters written to them. Principles:
I. In a case of contributory negligence the plaintiff through their own negligence, contributes to the harm he/she suffers.
Principles: II. Voluntary intoxication is no defence.
I. An offer lapses on the expiry of time, if any, fixed for its acceptance. III. A defendant is liable for the damages suffered by the plaintiff that could have been reasonable foreseen by the defendant.
II. The date on which the option of acceptance is exercised by the party is to be ascertained from the date on which acceptance is put in Facts: Vijay had gone for an office party after work where he had consumed several glasses of a rare variety of vodka on
transmission. the insistence of his boss. It was well past midnight when Vijay decided to return home from the party. He was in a state of
26. Facts: A sends a letter to B on 15 th January offering to sell his house for Rs. 24 lakhs. He stipulates that in case B is intoxication and was not quite aware of his surroundings. Sukanya was an upcoming investigative journalist who was on
interested, he must accept the offer by 30th January. B sends a letter to A accepting the proposal on 24 th January. Due to her way back home after a late night reporting. Her house was hardly ten minutes away from her office so she decided to
weather conditions, the mail gets delayed and reaches A on 3 rd Februar . A sa s he s not bound b the acceptance. B walk back to her house. While approaching the main crossing, Vijay sees a flash of light and in the other instant he sees a
sues A. dog coming towards his car. In an attempt to save the dog, he swerved to the right and he hit Sukanya who was talking on
(a) A is not bound by the acceptance as it reached him later than the time stipulated for acceptance. the phone. Sukanya was knocked into the air and was flung a few meters ahead onto the street. Due to the impact of the
(b) A is not bound by the acceptance as he could not be sure that the letter was actually put in transmission before the car, she died on the spot.
30th.
(c) A is not bound by the acceptance there has been lapse of the offer. 30. Did Sukanya in any manner whatsoever contributed to her death?
(d) None of the above. (a) Yes. As Sukanya was talking on the phone while walking, she was partly responsible for her own death.
(b) No. Sukanya was just talking on the phone. It does not imply that she was negligent while walking back home.
Principle: If someone willingly places themselves in a position where harm might result, knowing that some degree of harm might (c) No. There is nothing given in the facts to indicate that she was not aware of her surroundings and had hence
result, they are not able to bring a claim against the other party in tort. contributed to her death.
27. Facts: A is a cricket fan. He bought aticket for a match being played by his favorite team Mumbai Indians. During the (d) Yes. Sukanya should have been vigilant while walking back so late at night.
match, due to a six hit by Sachin Tendulkar, the ball hits him on the nose. He gets severely injured. He sues BCCI and Answers:
Mumbai Indians. (a) A and B
(a) He shall succeed as the defendants are responsible for the injury he suffered. (b) D
(b) He shall succeed because he was a victim of a wrongful act. (c) B and C
(c) He shall not succeed because on buying tickets, he voluntarily placed himself in a situation where the ball could (d) None of the above
hurt him.
(d) He shall not succeed as such injuries are a part of the game. 31. Was Sukan a s death a foreseeable consequence of Vija s actions?
(a) No. He was trying to save the dog and hence he did not cause the accident deliberately.
Principles:
(b) Yes. Vijay was in an inebriated state while driving. The accident was a reasonably foreseeable consequence of his
I. The active concealment of a fact by one having knowledge or belief of the fact is fraud.
condition.

©LegalEdge Tutorials Page 27 of 60 ©LegalEdge Tutorials Page 28 of 60


Replication or other unauthorized use of this material is prohibited by the copyright laws of India Replication or other unauthorized use of this material is prohibited by the copyright laws of India
LE Prep Assist LE Prep Assist
(c) No. Vijay is not liable as he could not see in the dark at night. One day, Bubbles ran out cash to buy cocaine, so she sold off Buttercup's white gold bracelet, in exchange for drugs. She
(d) Yes. Vijay lost control of the car while crossing the road due to which Sukanya lost her life. thought she'd replace Buttercup's bracelet as soon as she received money from home and that she'd never find out, since
she was on a road trip with her other friends for a week.
Answers:
(a) A and C However, Buttercup returned sooner than expected and on discovering her missing bracelet, she realized what had
(b) B happened and immediately called the cops. On inspection, they found large quantities of cocaine in Buttercup's cupboard
(c) B and D and arrested her for illegal possession of drugs, which is a non-cognizable offence. When in prison, the police offered
(d) D Buttercup a lighter sentence if she confessed to the crime. She confessed the same in Court.

32. Can Vijay plead the defence of involuntary intoxication? 36. Is Bubbles liable for theft?
(a) Yes. He drank onl on his boss s insistence. (a) Yes, since she took away the bracelet with the intention of buying drugs
(b) No. Vijay got drunk voluntarily. No one forced him to have alcohol. (b) No, since she intended to replace it as soon as possible
(c) Yes. Vijay had to drink under peer pressure. (c) No, since Buttercup was not in possession of the bracelet when she stole it
Answers: (d) Yes, because she didn t intend to bu him a new bracelet
(a) A
(b) C Principle: All States shall refrain from the use of threat or force against the territorial integrity or political independence of any other
(c) B State.
(d) None of the above 37. Facts: There was a massive and violent public uprising in the State of Tulli that threatened the existing government.
The Prime Minister of Tulli, upon instructions from the King of Tulli, asked the President of the State of Gulan to assist
Principles: (Q 33 to Q 35): them by sending military aid so that the massive violent uprising could be suppressed. Military aid was sent as per the
I. In case of fraud, the defendant knows that the statement is false or honestly believes it to be true. request. The leader behind the uprising,Khal Bali, was arrested and tried. Bali took the case to the World Supreme
II. To constitute fraud, the defendant should make a positive false statement. Mere silence as to the falsity is no fraud. Court, arguing that Gulan had interfered in the internal political affairs of the nation of Tulli. Will Khal Bali succeed?
III. To constitute fraud, it has to be shown that the statement was made with an intention to deceive.
Facts: X was a candidate for a graduate level examination of Y University. When the first attendance list came out, he had (a) No, as Gulan did not interfere in the internal political matters of Tulli, but merely provided aid, on request
the enough attendance required by the university to be eligible to sit for the examination. He filled the examination form (b) No, as the State of Gulan merely assisted in the suppression of a situation that had gotten out of hand
and got the admit card. In the revised list which came later, he was short of attendance required to sit for the examination. (c) No, as an ordinary citizen, Khal Bali will not be able to bring his case in front of the World Supreme Court
The administration did not scrutinize this fact and went ahead with the examination in which X also appeared. Later it was (d) Yes, as the uprising was a political event, and the State of Gulan had interfered in it, by the use of military force
found out that X did not have the requisite attendance. The university wants to sue him for fraud.
Principles:
33. Did X make a false statement with a belief in its falsity? I. A mere right to sue is personal to the party aggrieved and is not transferable for the breach of a contract.
(a) No. X did not make any statement. He simply stayed silent on the matter. II. In case a property is transferred along with the right to recover damages or compensation in respect of that property, such transfer
(b) No. X did not make any statement thinking it to be false. is valid.
(c) Yes. X did not say anything when the revised attendance list came out. 38. Facts: Atmanand agrees to sell 15 porcelain dolls to Bhakti on a future date. Bhakti transfers her interest to Cecilia. A
(d) Yes. X did not raise the issue when the University administration failed to check the facts. few days later, Atmanand breaches the contract, and Cecilia files a suit against him to recover damages. Will she
succeed?
34. Can X s silence be construed as fraud?
(a) Yes. He led the University Administration into believing that he had enough attendance to sit for the exams. (a) No, as the right to sue vested with Bhakti
(b) No. He remained silent on the matter which is an essential element of fraud and hence it will not constitute fraud. (b) No, as the right to sue is a personal right and there existed no contract between Atmanand and Cecilia
(c) Yes. His silence on the matter misrepresented facts in different light in front of the University administration. (c) Yes, as the right to recover compensation has been vested in Cecilia by Bhakti
(d) No. There was no intention to deceive the University on the matter. (d) Yes, as Atmanand failed to supply the porcelain dolls in time

35. Can it be said that X had intention to deceive the university? Principles:
(a) Yes. X led the University into believing that he could sit through the examinations. I. The Memorandum of Association of a company contains the fundamental conditions upon which the company is allowed to be
(b) Yes. X did not report the revised attendance list to the University when it showed his low attendance. incorporated. It sets forth the area of operation of the company and its objects.
(c) No. X did not make an statement whatsoever and hence there can be no intention to deceive on X s part. II. A company may alter its objects only in so far as the change is necessary to carry on the business of the company in a more
(d) No. There was no intention to deceive on X s behalf. economically or efficient manner, or attain the main purpose of the company by new or improved means, or enlarge the local area
of operation of the company, or carry some business which, under the existing circumstances, may be combined with the company s
Principle: A person is liable for theft if he takes away any movable or immovable property from the possession or ownership of another business, provided that the new business is not inconsistent with or contradictory to the purpose of the existing business.
person with a dishonest intention or for an illegal purpose. 39. Facts: Green Gro is a company engaged in the business of producing natural and organic varieties of exotic fruits. The
Facts: Bubbles and Buttercup studied together in school, and for graduation, they both joined the same college and shared company sought to amend its objects for purposes of undertaking the business of manufacturing water-soluble
a flat. Bubbles also became good friends with Blossom, who also studied with her and began spending a lot of time together. pesticides for farmers. Which of the following is true?
Blossom was addicted to cocaine and in her company, Bubbles also became a regular at it. Bubbles used to stash all her
drugs in Buttercup's cupboard. (a) Green Gro can amend its objects to undertake a pesticide manufacturing business
(b) Green Gro can't amend its objects as the proposed business is inconsistent with the current one
(c) Green Gro can amend its objects, provided that it changes its name as well, since Green Gro is misleading
©LegalEdge Tutorials Page 29 of 60 ©LegalEdge Tutorials Page 30 of 60
Replication or other unauthorized use of this material is prohibited by the copyright laws of India Replication or other unauthorized use of this material is prohibited by the copyright laws of India
LE Prep Assist LE Prep Assist
(d) Green Gro can amend its objects and set up a new company as that will enlarge its area of operation (d) Mishra's suit will not succeed because the government's action was not arbitrary.

Principles: Principle; A master can be held vicariously liable for the wrongs of his servant if the wrongs have been committed in the course of the
I. Article 48A of the Indian Constitution declares that "The State shall endeavor to protect and improve the environment and to latter's duty.
safeguard the forests and wildlife of the countr . 45. Facts: A goes to B's house for dinner at his invitation. A presses the button to call for the lift and when the door opens, steps
II. Article 51A (g) of the Constitution imposes a responsibility on every citizen to protect and improve the natural environment through. But the lift is not there and A is severely injured. It later turns out that the lift hasn't yet been installed. In which of these
including forests, lakes, rivers and wildlife, and to have compassion for living creatures. circumstances can A sue the owner of the building?
III. Article 19(1)(g) of the Constitution bestows upon the citizens the fundamental right to carry on any trade or business, subject to (a) If the lift's maintenance was under the care of a team of employees hired and paid by the owner.
reasonable restrictions. (b) If the maintenance of the building was contracted out to a firm which specialises in looking after such buildings.
40. Facts: Marty ran a factory that manufactured aerated drinks. His factory discharged untreated effluents into the lake (c) The maintenance of the lift was in the hands of an association of the residents of the building.
nearby. The Municipal Corporation sent a notice to employ safe disposal methods or they would be forced to shut (d) The lift was a private lift special y installed and maintained by B for the private use of his family and guests.
down his factory. Marty paid no heed to the notice and the Municipal Corporation sealed his factory. Marty went to
Court and claimed that this was against his fundamental right to carry on any trade or business. Decide. Principle: Nothing is an offence which is done by a person who is, or who by reason of a mistake of fact and not by reason of a mistake
(a) The Municipal Corporation cannot violate the Fundamental Right to carry on trade or business of law in good faith believes himself to be, bound by law to do it.
(b) The Municipal Corporation should pay damages to Marty for the loss caused by shutting down his factory Explanation: This principle is applicable not only to Government servants but civilians also.
(c) The Municipal Corporation imposed a reasonable restriction on Marty right to carry on trade and business 46. Facts: A who is a court officer was ordered by that court to arrest Y, after due inquiry, believing Z to be Y, arrests Z.
(d) Marty is an irresponsible individual as he did not heed the warning of the Municipal Corporation What offence has A committed?
(a) No, A has not committed any offence but he may be liable for negligence.
Principle: An agreement to oust the jurisdiction of courts is void ab initio. (b) A has committed an offence by illegally arresting Z.
Explanation: Jurisdiction refers to the ability and authority to entertain cases. Any agreement, unenforceable in a court of law, is void. (c) A has committed an offence only if it is proved that Z is innocent of all other crimes.
41. Facts: Monda Automobiles entered into an agreement of sale with Zero Tyres under which Monda Automobiles (d) A has not committed any offence.
decided to sell its shareholding in Ronda Automobiles to Zero Tyres. As per the agreement, if there arose any dispute
pertaining to the valuation of the shareholding, it would be compulsorily referred to an arbitration tribunal and neither Principle: If a dangerous thing is brought on to the land for non-natural use of the land, and the dangerous thing escapes, then the
party would approach the court for any matter in connection with the award of the tribunal until a period of 3 months person who brought it on to the land is strictly liable for any damage caused by the dangerous thing.
had elapsed between the date of award and approaching the court. Which of the following is correct? Explanation: If the damage is caused by the escape due to the plaintiff's own default, then the person who had brought the dangerous
(a) The agreement entered into is void ab initio since it prohibits the court from interfering thing onto his property is not liable. Plaintiff is the person who complains against another in the court.
(b) The agreement is void since it is against the principle of judicial primacy 47. Facts: Jamukha has the habit of bringing exotic animals on to his land for his entertainment. His neighbor Chingiz
(c) The agreement is valid since the parties entered into it with their free consent deliberately leaves the gate of Jamukha's garden open, knowing that the lion and cheetahs which are enclosed there
(d) The agreement is not void since it does not eject the jurisdiction of the courts will escape. The lion immediately rushes out of the gate and mauls Chingiz. He sues Jamukha.
(a) Jamukha is not liable because the escape and damage were all caused by act of Chingiz only.
Principle: A person who owns a place is responsible for its maintenance. (b) Jamukha is strictly liable.
42. Facts: A family goes to a South Indian restaurant for dinner, and while enjoying the delicious food, the fan directly (c) Jamukha is not strictly liable but will be liable if negligence on his part is proved.
above their table falls on their son, thereby injuring him. The family sues the restaurant owner for the same. The owner (d) Chingiz is liable to the extent of the injuries he expected the animals to cause.
blames it on the electrician who fixed the fan. Can the owner escape liability?
(a) Yes, it was the electrician s fault, as he was responsible for all the technical maintenance. 48. Who among the following have the Right to Vote in the elections to both the Lok Sabha and the Rajya Sabha?
(b) No, although it was the electrician s fault, the owner should still have looked into the matter. (a) Elected members of the Lowers House of the Parliament
(c) Yes, according to the principle of Volenti Non Fit Injuria . (b) Elected Members of the Upper House of the Parliament
(d) No, the owner should treat the family with free food for a year. (c) Elected Members of the Upper House of the State Legislature
(d) Elected Members of the Lower House of the State Legislature
Principle: The right of self-defence in no case extends to inflicting more harm than it is necessary to inflict for the purpose of defence.
43. Facts: There was a fight between two groups. Ram Singh fired a shot, killing one of the assailants on the spot. Ram 49. The authorization for the withdrawal of funds from the Consolidated Fund of India must come from:
himself was found with nine injuries on his body and the diagnosis revealed that he could not have survived if was (a) The President of India
injured anymore. He was arrested by the police and convicted for murder. Ram Singh claimed the right of self-defence. (b) The Parliament of India
(a) Ram Singh is liable for intentionally killing a person. (c) The Prime Minister of India
(b) Ram Singh is not liable for killing. (d) The Union Finance Minister
(c) Ram Singh can exercise self-defence.
(d) Ram Singh cannot claim self-defence since he extended more harm than necessary. 50. The test of creamy layer in reservation policy in India is not applicable to
(a) SC
Principle: All government contracts are subject to non-arbitrariness, principles of equity and natural justice. (b) ST
44. Facts: Mishra submitted a tender for obtaining a contract with the Bangalore Development Authority to construct a new ring (c) OBC
road in Bangalore. When the tenders were opened it was found that his tender was the lowest bid. However, the contract was (d) Both (A) and (B)
awarded to another company without giving any reasons, Mishra challenged the awarding of the contract in the court. Decide.
(a) Mishra's suit will not succeed since the government has the right to decide whom to award the contract.
(b) Mishra's suit will succeed since the government's action was arbitrary and violative of equity.
(c) Mishra's suit will succeed since the government did not choose the tender which was the lowest.

©LegalEdge Tutorials Page 31 of 60 ©LegalEdge Tutorials Page 32 of 60


Replication or other unauthorized use of this material is prohibited by the copyright laws of India Replication or other unauthorized use of this material is prohibited by the copyright laws of India
LE Prep Assist LE Prep Assist

Prep Test Mathematics 12. A man purchases a certain no. of apple at 5 per rupee and same no. at 4 per rupee. He mixes them together and sells
them at 4 per rupee. What is his gain or loss%?
Time: 15 min. No. of Questions: 20 (a) Gain 20 % (b) Gain 11.11% (c) Loss 11.11% (d) Loss 20 %
1. Aadesh bought a combined total of 20 monitors and printers. He marked up the monitors by 20% on CP while each
printer was marked up by Rs. 2000. He was able to sell 75% of the monitors and 2 printers and make a profit of Rs. 13. A trader allows a Discount of 5% for cash payment. How much approx % above cost price must he mark his goods
49,000. The remaining monitors and 3 printers could not be sold by him. Find his overall profit or loss if he gets no to make a profit of 10%?
return on unsold items and it is known that a printer costs 50% of a monitor. (a) 8.9% (b) 10% (c) 12.75% (d) 15.8%
(a) Loss of Rs. 48,500 (b) Loss of Rs. 21,000 (c) Loss of Rs. 41,000 (d) Data Inadequate
14. If selling price is doubled, the profit triples. Find the profit percent?
2. A rickshaw dealer buys 30 rickshaws for Rs.4725. Of these, 8 are four seaters and rest are two seaters. At what price (a) 100% (b) 116.67% (c) 200% (d) 300%
must he sell the four seaters so that if he sells the two seaters at 3/4th of this price, he makes a profit of 40% on his
outlay.
(a) Rs. 180 (b) Rs. 270 (c) Rs. 360 (d) Rs. 450 15. The percentage profit earned by selling an article for Rs. 1920 is equal to the percentage loss incurred by selling the
same article for Rs. 1280. At what price should the article be sold to make 25% profit?
3. Ritesh bought 25 washing machines and microwave ovens for Rs. 2,05,000. He sold 80% of the washing machines (a) 2200 (b) 2400 (d) 2500 (d) 2000
and 12 microwaves ovens for a profit of Rs 40,000. Each washing machine was marked up by 20% over cost and
each microwave oven was sold at a profit of Rs. 2,000. The remaining washing machines and 3 microwave ovens 16. Abhishek purchased 20 dozens of toys at the rate of Rs. 375 per dozen. He sold each one of them at the rate of Rs.
could not be sold. What is Raghav s overall profit/loss? 33. What was his percentage profit?
(a) Rs. 1000 profit (b) Rs. 2500 loss (c) Rs. 1000 loss (d) Cannot be determined (a) 5.4 (b) 5.6 (c) 6.5 (d) 4.5

4. A flat and a piece of land were bought by two friends Tarun and Varun respectively at prices of Rs. 2Lakh and Rs. 17. Some articles were bought at 6 articles for Rs. 5 and sold at 5 articles for Rs. 6. Gain percent is:
2.2 Lakh. The price of the flat rises by 20% every year and that of land by 10% every year. After two years, they (a) 33.33% (b) 66.66% (c) 44% (d) 50%
decided to exchange their possessions. What is approx. percentage gain of the gainer?
(a) 7.56% (b) 6.36% (c) 4.39% (d) None of these 18. On selling 17 toys at Rs. 720, there is a loss equal to the cost price of 5 toys . The cost price of a Toy is:
(a) Rs. 50 (b) Rs. 60 (c) Rs. 65 (d) Rs. 70
5. Sunil calculates his profit percentage on the selling price whereas Sujeet calculates his profit on the cost price. They
find that the difference of their profits is Rs. 900. If the selling price of both of them are the same, and Sunil gets 50% 19. A shopkeeper sells some articles at the profit of 25% on the original price. What is the exact amount of profit? To
profit and Sujeet gets 40% profit, then find their selling price. find the answer, which of the following information given in Statements I and II is/are Sufficent?
(a) Rs 4200 (b) Rs 4500 (c) Rs 4000 (d) Rs 4800 I. Sale price of the article
II. Number of articles sold
6. A reduction of 10% in the price of salt enables a person to buy 2 kg more for Rs.180. Find the reduced and the (a) Only I is sufficient. (b) Only II is sufficient. (c) Both I & II are sufficient. (d) Either I or II are sufficient.
original price per kg of salt respectively.
(a) Rs 10, Rs 9 (b) Rs 9, Rs 10 (c) Rs 18, Rs 20 (d) Rs 20, Rs 18
20. A man purchases 10 Cows at Rs. 3000 each. 1 Cow died. He sold 2 Cows at 5% loss, at what rate he should sale the
7. A person sold his watch for Rs. 24. If the percentage of his loss was equal to the cost price , then the watch would remaining Cows, so as to gain a Profit of 10 % on the total Cost?
have cost him (a) Rs.4000 (b) Rs.3000 (c) Rs.3900 (d) Rs. 4500
(a) Rs. 40, 60 (b) Rs. 60,50 (c) Rs. 50,50 (d) None of these

8. A man buys two horses for Rs. 1550. He sells one so as to lose 23% and other so as to gain 27%. On the whole
transaction he neither gains nor loses. What does each horse costs?
(a) 807,743 (b) 817,733 (c) 827,723 (d) 837,713

9. An orange vendor makes a profit of 20% by selling oranges at a certain price. If he charges Rs. 1.2 higher per orange
he would gain 40%. Find the original price at which he sold an orange.
(a) Rs. 3 (b) Rs. 12 (c) Rs. 4.8 (d) None of these

10. After selling a watch, Shyam found that he had made a loss of 10%. He also found that had he sold it for Rs.27
more, he would have made a profit of 5%. The actual initial loss was what percentage of the profit earned, had he
sold the watch for a 5% profit?
(a) 23% (b) 150% (c) 200% (d) 180%

11. 1/3 of a commodity is sold at 15% profit, ¼ is sold at 20% profit and the rest at 24% profit. If the Total profit is Rs.
80 is earned then find the value of commodity?
(a) 350 (b) 410 (c) 400 (d) 300

©LegalEdge Tutorials Page 33 of 60 ©LegalEdge Tutorials Page 34 of 60


Replication or other unauthorized use of this material is prohibited by the copyright laws of India Replication or other unauthorized use of this material is prohibited by the copyright laws of India
LE Prep Assist LE Prep Assist
19. Which countr s President signed the Asia
1. According to ISRO,the first manned mission to (c) Australia (d) Switzerland Reassurance Initiative Act? 29. What is the theme for the 106th Indian Science
space will be sent by which year? (a) Russia (b) USA Congress (ISC-2019)?
(a) 2030 (b) 2025 10. Which state has topped the Ease of Doing Business (c) UK (d) Ukraine (a) Future: Artificial Intelligence In India
(c) 2021 (d) 2022 Rankings for Indian states? (b) Future: India-Science and Technology
(a) Maharashtra (b) Andhra Pradesh 20. Who has become become the first women (c) India-Technology and Internet of Things
2. Who among the following has sworn in as (c) West Bengal (d) Delhi amputee to climb Mount Vinson (Highest peak (d) Science of Internet and Technology
Bra il s new President? of Antarctica?
(a) Luiz Inácio Lula da Silva 11. Sultan Muhammad V is/was the King of _______. (a) Arunima Sinha 30. Ramakant Achrekar passed away following a
(b) Jair Bolsonaro (a) Mauritius (b) Malaysia (b) Santosh Yadav heart attack. He was a veteran________.
(c) Michel Temer (c) Spain (d) Nepal (c) Premlata Yadav (a) Dance teacher (b) Cartoonist
(d) Dilma Rousseff (d) Malavath Purna (c) Journalist (d) Cricket coach
12. Who is India s Chief Economic Advisor?
3. Which campaign was launched by the (a) Arvind Subramanian 21. When is World Braille Day observed? 31. Veer Savarkar International Airport has been
government to celebrate the exceptional (b) C.N.R. Rao (a) Jan 6 (b) Jan 5 declared as an authorized Immigration Check
achievements of women who have influenced (c) T.S. Krishnamurthi (c) Jan 4 (d) Jan 1 post for entry into/exit from India. This airport
and impacted society through social media? (d) Krishnamurthy Subramanian is located at_______.
(a) Strong India,Strong Women Campaign 22. Who will head the committee that has been (a) Kanyakumari (b) Ross Island
(b) India Women Campaign 13. Who was recently sworn in as Supreme Court formed to rejuvenate Micro, Small and Medium (c) Lakshadweep (d) Port Blair
(c) Web Wonder Women Campaign Judges by Chief Justice of India Ranjan Gogoi? Enterprises?
(d) India Ladies Campaign (a) Ranjan Das and CV Singh (a) Ranjan Das 32. Telangana has got its first independent high court
(b) Dinesh Maheshwari and Sanjiv Khanna (b) Praveen Garg now. Who has sworn in as the first Chief Justice of
4. Who will head the panel that has been (c) Abhirup Bhattacharya and CK Khanna (c) Subhash Chandra Khuntia the Telangana High Court?
constituted to hear the title suit in the Ram (d) Sanjiv Khanna and CK Khanna (d) Upendra Kumar Sinha (a) Justice Shri Krishna Ayyar
Janmabhoomi-Babri Masjid Ayodhya case? (b) Justice BT Ramamurthi
(a) K Joseph 14. Who will head the 11 member working group 23. Veteran coach Ramakant Achrekar passed away (c) Justice TBN Radhakrishnan
(b) Ranjan Gogoi set up to go into the concept of Group recently.He was the coach of which sports? (d) Justice BN Kumaraswami
(c) Sunil Mehta Insolvenc and suggest a suitable framework? (a) Kabaddi (b) Tennis
(d) Prashant Bhushan (a) UK Sinha (c) Football (d) Cricket 33. Which tropical storm recently hitted Philippines
(b) Ajay Tyagi on Christmas Day?
5. Which state s Polavaram Project entered (c) Subhash Chandra Khuntia 24. Who will head the committee that has been (a) Huva (b) Usman
Guinness Book of World Records by completing (d) M Nageswara Rao setup for effective implementation of real estate (c) Kwacha (d) Katrina
32,315.5 Cubic meter of concrete works in a law RERA?
span of 24 hours? 15. Andry Rajoelina was recently sworn in as
(a) Shiv Das Meena (b) Praveen Kumar 34. Who was appointed as new chairman of
(a) Karnataka (b) Telangana President of which country?
(c) Ekta Singh (d) Sudhakar Maurya Railway Board?
(c) Andhra Pradesh (d) Sikkim (a) Ukraine (b) Madagascar
(a) VK Yadav
(c) Brazil (d) Uganda
25. In which state, around 50 lakh women formed a (b) Ashwani Lohani
6. According to WHO, which country accounts 620 km long Women s Wall? (c) Subhash Chandra Khuntia
16. Which countr s airport to be renamed after
more than half leprosy cases globally? (a) Mizoram (b) Karnataka (d) Praveen Garg
boxing legend Muhammad Ali?
(a) China (b) Nepal (c) Kerala (d) Sikkim
(a) UK (b) Russia
(c) Pakistan (d) India 35. Who was appointed as new new Chief
(c) USA (d) Pakistan
26. Which organi ation launched Samwad with Information Commissioner?
7. The world s longest 3D-printed concrete
17. Who became the first tycoon to be charged Students on New Year Day? (a) Yashvardhan Kumar Sinha
pedestrian bridge opened in which country?
under a new anti fraud law? (a) SIDBI (b) RBI (b) Sudhir Bhargava
(a) UK (b) USA
(a) Sanjay Bhattra (c) NABARD (d) ISRO (c) Vanaja N.Sarna
(c) Japan (d) China
(b) Mehul Choksi (d) Suresh Chandra
8. Which countr s government declared (c) Nirav Modi 27. Which two countries recently officially quit the
sugarcane juice as the national drink of the (d) Vijay Mallya U.N. s educational, scientific and cultural 36. Who will head the panel that has been setup to
country? agency (UNESCO)? suggest parameters for an equitable distribution
(a) India (b) Nepal 18. Which tropical storm recently hitted and made (a) Israel and Iran (b) USA and Israel of OBC reservation?
(c) Pakistan (d) Sri Lanka landfall in Thailand? (c) India and USA (d) Iraq and Japan (a) Arundhati Bhateja
(a) Hudhud (b) Nilofar (b) Praveen Kumar
9. Which of the following teams has won the (c) Katrina (d) Pabuk 28. Which country recently introduced GAFA tax? (c) TS Balakrishnan
Hopman Cup? (a) France (b) Russia (d) G Rohini
(a) U.S.A (b) U.K. (c) India (d) USA
©LegalEdge Tutorials Page 35 of 60 ©LegalEdge Tutorials Page 36 of 60
Replication or other unauthorized use of this material is prohibited by the copyright laws of India Replication or other unauthorized use of this material is prohibited by the copyright laws of India
LE Prep Assist LE Prep Assist
37. Which state s Police facebook page was selected (c) BSF (d) Dogra Regiment 55. India has recently signed a long-term contract 64. Who has been bestowed with Sansad Ratna
b Microsoft , to stud the organisation s new with ________ for supply of uranium ore Award for his distinguished performance as a
age media interactions? 46. India s fastest indigenous train which will run concentrates to widen its source base for the parliamentarian under Jury Committee Special
(a) Kerala (b) Karnataka from Delhi to Varanasi has been named as nuclear fuel. Award category?
(c) Mizoram (d) Bihar ____________. (a) Maldives (b) The UAE (a) Anurag Thakur (b) Sharad Pawar
(a) Vande Bharat Express (c) Australia (d) Uzbekistan (c) Bhagwant Mann (d) Narendra Modi
38. Which Chinese space probe is moving into (b) Vande Mataram Express
position to land on the dark side of the moon (c) Bharat Mata Express 56. Who has been appointed as the Director 65. Which country has banned the use of Indian
for the first time? (d) Ek Bharat Express General (Investigation) in the National Human currency notes of Rs 2,000, Rs 500 and Rs 200
(a) Yuan 2 Rights Commission. denominations?
(b) Chang e 4 47. Who among the following won the Australian (a) Harish Kumar Brijwal (a) Sri Lanka (b) Bhutan
(c) Cixco 3 Open 2019, Men s Singles tennis title? (b) Ranveer Singh Maurya (c) Nepal (d) Mauritius
(d) Crim 5 (a) Rafael Nadal (b) Henri Kontinen (c) Kumar Gaurav Singh
(c) Nicolas Mahut (d) Novak Djokovic (d) Prabhat Singh 66. Who is the Chief Guest of the 15th Pravasi
39. Which countr announced 2019 as the Year of Bharatiya Diwas Convention?
Active Investment and Social Development ? 48. Who among the following won the Australian 57. The Swedish Parliament approved _________ as (a) Satya Nadella
(a) Afghanistan (b) Turkmenistan Open 2019, Women s Singles tennis title? Prime Minister for a second four-year term. (b) Baljeet Singh
(c) Uzbekistan (d) Uganda (a) Samantha Stosur (b) Naomi Osaka (a) Scott Morrison (b) Gaston Browne (c) Kanwaljit Singh Bakshi
(c) Petra Kvitova (d) Zhang Shuai (c) Donald Tusk (d) Stefan Lofven (d) Pravind Jugnauth
40. Sultan Abdullah Sultan Ahmad Shah has been
elected as _______ new king by the members of 49. Name the former Defence Minister, who passed 58. UNESCO has named ________ as the World 67. India ranks _____ on the global talent
the countr s ro al families. away in Delhi at the age of 88 recently. Capital of Architecture for 2020. competitive index?
(a) Brunei's (b) Cambodia's (a) George Fernandes (b) Jaswant Singh (a) London (b) Paris (a) 80th (b) 82 nd
(c) Singapore's (d) Mala sia s (c) Pranab Mukherjee (d) Manohar Parrikar (c) Rio de Janeiro (d) Berlin (c) 75th (d) 61 st

41. Krishna Sobti has passed away in Delhi. She 50. A 5.1-km long cable-sta ed Atal Setu on the 59. Defence Minister Nirmala Sitharaman 68. Which state government has decided to cater
was a well known_______. Mandovi river in- inaugurated the 426 meters long Diffo Bridge. It 5% reservation each for Kapu community and
(a) Sports person (b) Activist (a) Gujarat (b) Maharashtra is built over _______. the economically backward classes (EBCs)
(c) Writer (d) Freedom Fighter (c) Goa (d) Kerala (a) Kali River (b) Brahmaputra River among the forward castes?
(c) Son River (d) Chipu River (a) Andhra Pradesh (b) Manipur
42. The Defence Research and Development 51. Who has become the first Hindu woman to (c) Meghalaya (d) Telangana
Organisation (DRDO) has successfully test fired have been appointed as a civil judge in Pakistan 60. Which of the following city hosted the 15th
Long Range Surface to Air Missile (LR-SAM) after passing an examination for induction of edition of Pravasi Bhartiya Divas? 69. Ten ears after 26/11 , the Indian Nav
from naval warship ________. judicial officers? (a) Varanasi (b) Jaipur commenced the largest coastal defence exercise
(a) INS Kohasa (b) INS Chennai (a) Aditi Singhal (b) Swati Jha (c) Jodhpur (d) Nagpur off the Indian coast. What is the name of the
(c) INS Vikrant (d) INS Khukri (c) Ruchi Srivastava (d) Suman Kumari exercise?
61. What is the theme of the Pravasi Bhartiya Divas (a) The Sea Voyage (b) VAJRA PRAHAR
43. The government has instituted an annual award 52. Which country has replaced Japan as the 2019? (c) Yudh Abhyas (d) SEA VIGIL
titled Subhash Chandra Bose Aapda world s 2nd largest steel producing country, (a) Indian Diaspora and Developing India
Prabandhan Puraskar. This year _______has according to the World Steel Association? (b) New India and Role of Indian Diaspora 70. 'Global Risk Report' is released annually by
been selected for the Award. (a) China (b) India (c) Building New India which of the following organisation?
(a) Garhwal regiment of Indian Army (c) Brazil (d) Germany (d) Role of Indian Diaspora in building New India (a) IMF
(b) 17th Battalion of BSF (b) World Bank
(c) 7th Battalion of NDRF 53. Which country will host the 14th session of the 62. The Insolvency and Bankruptcy Board of India (c) United Nations
(d) 8th Battalion of NDRF Conference of Parties to the United Nations (IBBI) has set up an 11-member working group (d) World Economic Forum
Convention to Combat Desertification under the Chairmanship of____________.
44. National Voters Day is celebrated every year on (UNCC(d)? (a) R S Sharma (b) Shaktikanta Das 71. Fitch Rating company Headquarters is in
_______________. (a) Japan (b) China (c) Ajay Tyagi (d) UK Sinha ________.
(a) 24 January (b) 23 January (c) India (d) Sri Lanka (a) New York (b) London
(c) 27 January (d) 25 January 63. Who has been sworn in as Madagascar (c) Gurugram (d) Washington DC
54. ________will be the site of the countr s second President?
45. The all-women ________ contingent created defence innovation hub after Coimbatore in (a) Donald Tusk (b) Stefan 72. A ministerial panel headed by ___________ will
history by participating for the first time in a Tamil Nadu. Lofven examine uniformity of taxation on lottery under
Republic Day parade. (a) Varanasi (b) Nashik (c) Andry Rajoelina (d) Gaston Browne GST and other issues arising out of it.
(a) Assam Rifles (b) Garhwal Rifles (c) Dehradun (d) Pathankot (a) Krishna Byre Gowda
©LegalEdge Tutorials Page 37 of 60 ©LegalEdge Tutorials Page 38 of 60
Replication or other unauthorized use of this material is prohibited by the copyright laws of India Replication or other unauthorized use of this material is prohibited by the copyright laws of India
LE Prep Assist LE Prep Assist
(b) Sudhir Mungantiwar 80. Which country will host the 2019 Africa Cup of the following? (c) APJ Abdul Kalam Island
(c) TM Thomas Isaac Nations? (a) Garibrath Express (d) Atal Dweep
(d) Himanta Biswa Sarma (a) Ukraine (b) USA (b) Bharat Jai Express
(c) Egypt (d) Japan (c) Hindustan Express 98. When is World Hindi Day observed?
73. The Gandhi Peace Prize for the year 2018 has (d) Vande Bharat Express (a) Jan 15
been conferred on___________. 81. Who penned the book We Are Displaced:M (b) Jan 12
(a) Sulabh International Journey and Stories from Refugee Girls around 90. Recently,Competition Commission of (c) Jan 10
(b) Yohei Sasakawa the World ? India(CCI) gave its approval to (d) Jan 11
(c) Akshaya Patra Foundation (a) Vikram Seth (b) Jhumpa GlaxoSmithKline Consumer HealthCare for
(d) Ekal Abhiyan Trust Lahiri merger with which company? 99. Recently, fiction writer and essayist Krishna
(c) Arundhati Roy (d) Malala Yousafzai (a) Cipla Sobti passed away.She is the writer of which
74. Who has taken over as the chief coach for the (b) Lupin Limited language?
countr s female boxers? 82. A 2+2 Inter-Sessional meeting between the (c) Patanjali Ayurveda (a) Urdu (b) Hindi
(a) Yunfei Li United States and India took recently place in (d) Hindustan Unilever Limited (c) Tamil (d) Marathi
(b) Mohammed Ali Qamar ___________.
(c) Josie Gabuco (a) Washington DC (b) Delhi 91. The World Economic Forum annual meet began 100. The 2nd World Integrated Medicine Forum
(d) Pinki Rani (c) New York (d) Paris in which city? 2019 on the Regulation of Homeopathic Medical
(a) New York (b) Canberra Products held in which state?
75. The government has constituted a seven- 83. Which state has launched the One Famil One (c) Davos (d) London (a) Uttar Pradesh
member Group of Ministers (GoM) to boost the Job scheme, under which for ever famil one (b) Goa
92. Who was crowned champion of the Indonesia (c) Andhra Pradesh
real estate sector under the GST regime. This government job has been allotted?
Masters 2019 ? (d) Telangana
committee shall be headed by ____________. (a) Assam (b) Tripura
(a) PV Sindhu
(a) Sushil Modi (b) Nitin Patel (c) Sikkim (d) Uttarakhand
(b) Saina Nehwal 101. According to World Economic Survey,which
(c) Dinesh Arya (d) Manohar Parrikar
(c) Carolina Marin country have emerged as the biggest supporters
84. The National Youth Day is celebrated across the
(d) Ankita Raina of international aid?
76. The Parliament of Macedonia has passed the country on ___________.
(a) India (b) Pakistan
resolution to amend the constitution of the (a) 13th January (b) 16th January 93. Cyril Ramaphosa arrived in Delhi for his first (c) Nepal (d) SriLanka
country to rename it as the Republic of (c) 12th January (d) 18th January state visit ahead of the Republic day celebration
Northern Macedonia. What is the capital of this parade. He is the President of which country? 102. Who has been awarded with Saraswati Samman
country? 85. External Affairs Minister Sushma Swaraj (a) South Africa (b) Australia 2017?
(a) Praia (b) Tunis embarked recently on a two-day visit to (c) Netherlands (d) Ukraine (a) Sitanshu Yashaschandra
(c) Dili (d) Skopje Uzbekistan. What is the capital city of (b) Govind Chandra Pandey
Uzbekistan? 94. The celebration of 15th Pravasi Bhartiya Divas (c) Raghbir Bhola
77. Who has become the first north-eastern state to (a) Abu Dhabi (b) Muscat was commenced in which city? (d) Padma Sachdev
launch single emergenc number 112 ? (c) Bishkek (d) Tashkent (a) Mumbai (b) Chennai
(a) Assam (b) Meghalaya (c) Varanasi (d) Pune 103. What is the theme for National Voters Day
(c) Mizoram (d) Nagaland 86. ISRO Chairman Dr. K Sivan has announced that 2019?
the organization is well prepared to meet the 95. Which countr opened the ear s space (a) No Voter to be left behind
78. Former Defence Minister George Fernandes targets of launching Gaganyaan manned campaign by putting into orbit defence imaging (b) No one to be left behind
passed away. In 1976, he was arrested and tried in mission to outer space by _____________. satellite Microsoft R ? (c) Education and Empowerment of women
the infamous ______. (a) December 2024 (b) December 2023 (a) India (b) USA voters
(a) Jessica Lal Murder Case (c) December 2022 (d) December 2021 (c) Russia (d) Australia (d) Educate the voters
(b) Ayodhya Case 96. Wicketkeeper-batter Alyssa Healy has been
(c) Baroda dynamite case 87. Taiwan President Tsai Ing-wen has appointed 104. ISRO has successfully launched the PSLV C 44
named as ICC Women s T20I Pla er of the Year.
(d) 2G Spectrum Case ___________ as Prime Minister of Taiwan. carrying Kalamsat, a 10 cm cube weighing 1
She belongs to which of the following country?
(a) Ma Ying-jeou (b) Hu Weide kilogram communication satellite with a life
(a) Australia
79. The first National Consultation on Child (c) Yan Huiqing (d) Su Tseng-chang span of ________.
(b) England
Protection for 2019 was held in which city? (a) 12 months (b) 6 months
(c) South Africa
(a) Gurugram 88. What is the capital city of Venezuela? (c) 2 months (d) 2 years
(d) Scotland
(b) New Delhi (a) Boliver (b) Caracas
(c) Agra (c) Presa (d) Bolivia 105. The _______government has decided to form a
97. Neil islands of Andaman and Nicobar has been
(d) Mumbai State Tiger Protection Force to save the Big Cat
renamed as_________.
89. Recentl , Train 18 , India s first indigenous population in the State.
(a) Shaheed Dweep
semi-high speed train was renamed as which of (a) Bihar (b) Telangana
(b) Netaji Dweep
(c) Madhya Pradesh (d) Gujarat
©LegalEdge Tutorials Page 39 of 60 ©LegalEdge Tutorials Page 40 of 60
Replication or other unauthorized use of this material is prohibited by the copyright laws of India Replication or other unauthorized use of this material is prohibited by the copyright laws of India
LE Prep Assist LE Prep Assist
106. National Voters Day was first celebrated in 115. Who has been appointed as the director general 124. Who among the following has been named as 133. Who was conferred with the honorary rank of
________. in the National Human Rights Commission? the 2018 Confederation of African Football General of the Indian Army?
(a) 2014 (b) 1988 (a) Narsingh Rao (b) HL Dattu Player of the Year? (a) Bimal Verma
(c) 1950 (d) 2011 (c) Sunil Mehta (d) Prabhat Singh (a) Kalidou Koulibaly (b) Didier Drogba (b) SS Sangwan
(c) George Weah (d) Mohamed Salah (c) Jasbir Walia
107. Which state government has declared Pakke 116. Which countr launched world s first artificial (d) Purna Chandra Thapa
Paga Hornbill Festival (PPHF) as the State meteor shower? 125. IRCTC, the Railwa s subsidiary, will offer
Festival ? (a) Japan (b) China insurance of up to INR ______ for free for air 134. The Madras High Court has set up an expert
(a) Tripura (c) India (d) Russia passengers who book their tickets through its committee to find a solution to root out invasive
(b) Arunachal Pradesh portal. species of plants from the Western Ghats. The
(c) Sikkim 117. The Centre has taken a decision to make all new (a) 25 lakh (b) 10 lakh committee shall be headed by __________.
(d) Himachal Pradesh cars meet Euro 6 standards mandatory for (c) 50 lakh (d) 5 lakh (a) UK Sinha
initial launch from which year? (b) MP Bezbaruah
108. India s first-ever all-women political party the (a) Aug 1,2022 (b) April 1,2020 126. Recently,the parliament of Macedonia passed (c) D Raghavendra Nayyar
National Women s Part (NWP) was launched (c) March 1,2019 (d) April 1,2022 the resolution to amend the constitution of the (d) Cherukuri Raghavendra Babu
in which city? country to rename it as which of the following?
118. Defence Minister Smt Nirmala Sitharaman 135. Macedonia has renamed it as the Republic of
(a) Mumbai (b) Delhi (a) United States of Macedonia
inaugurated the 426.60 metre long Diffo Bridge Northern Macedonia. Who is the Prime
(c) Chennai (d) Pune (b) Republica Macedonia
in which state? Minister of this country?
(c) Republic of Southern Macedonia
109. Who became the fastest Indian to claim 100 (a) Arunachal Pradesh (a) Gjorge Ivanov (b) Nikola Gruevski
(d) Republic of Northern Macedonia
wickets in ODIs? (b) Nagaland (c) Zoran Zaev (d) Talat Xhaferi
(a) Yuzvendra Chahal (c) Sikkim 127. Which state became the first state to implement
(b) Mohammed Shami (d) Mizoram the 10 percent reservation for economically 136. Which of the following organisation releases
(c) Vijay Shankar weaker sections of the general category? India's Financial Stability Report?
119. Who has become the youngest grandmaster in
(d) Bhuvneshwar Kumar (a) Haryana (a) Finance Ministry
India and second in the world?
(b) Uttar Pradesh (b) State Bank of India
110. Which team created the world record for (a) Kartik Singh (b) Sachin Sen
(c) Andhra Pradesh (c) Small Industries Development Bank of India
highest run chase in Ranji Trophy? (c)DK Sharma (d) D Gukesh
(d) Gujarat (d) Reserve Bank of India
(a) UP (b) Assam
120. Who has become the first women trekking to
(c) Saurashtra (d) Kerala 128. In which cit ISRO is setting up a Human 137. France has introduced its own tax, GAFA Tax
Agasthyarkoodam peak, after the Kerala High
Space Flight Centre ? on global internet and technology firms. What
111. Which country has withdrawn from UN-backed Court lifted the ban on women s trekking?
(a) Delhi (b) Mumbai does F stands for in GAFA?
anti-corruption commission? (a) Premlata Agarwal
(c) Bengaluru (d) Lucknow (a) Fugitive (b) Farlando
(a) Ukraine (b) Uganda (b) Arunima Sinha
(c) Santosh Yadav (c) Foreign (d) Facebook
(c) Guatemala (d) Netherlands 129. When is National Youth Day celebrated?
(d) K Dhanya Sanal (a) 10 th Jan (b) 11th Jan
138. Up to what amount of existing debt the Reserve
112. Who will head a high level RBI committee (c) 12 th Jan (d) 13th Jan
121. Who has sworn in as Venezuela President for Bank has allowed a one-time restructuring for
assessing digitization of payments?
2nd Term? 130. Union Minister of Environment, Forest and the companies which have defaulted on
(a) Raghuram Rajan
(a) Nicolás Castro (b) Fidel Castro Climate Change Dr Harsh Vardhan launched payment, but the loans given to them have
(b)Vijay Shekhar Sharma
(c) Nicolás Maduro (d) Che Guevara the National Clean Air Programme aiming to continued to be classified as standard assets?
(c) Nandan Nilekani
reduce toxic particulate matter by which year? (a) Rs 20 crore (b) Rs 25 crore
(d) Bimal Jalan
122. Who among the following has been named Chef (a) 2025 (b) 2020 (c) Rs 15 crore (d) Rs 10 crore
de Mission for the 2020 Tokyo Olympics? (c) 2022 (d) 2024
113. Former Group President Jim Yong Kim
(a) Birendra Prasad Baishya 139. Who has been appointed the new Railway
resigned recently.He was the former president
(b) Punam Yadav 131. Who was appointed as new PM of Taiwan? Board Chairman?
of which organization?
(c) Venkat Rahul Ragala (a) Se Lee Chang (a) Vikram Bhargava
(a) AIIB (b) ADB
(d) Heena Sidhu (b) Tsai Lu (b) Sudhir Bhargava
(c) IMF (d) World Bank
(c) Tsai Ing Wen (c) V K Yadav
123. Which country topped the list of most powerful (d) Su Tseng Chang (d) V S Reddy
114. King Mohammed V became first monarch to
passports, according to the Henley Passport
resign before his tenure.He belongs to which 132. Who was appointed as new President of
Index? 140. Which country has withdrawn its membership
country? Venezuela?
(a) India (b) Japan from Organization of the Petroleum Exporting
(a) Uganda (b) Malaysia (a) Ole Sentsov (b) Scott Morrison
(c) Singapore (d) USA Countries (OPE(c)?
(c) Pakistan (d) Ukraine (c) Nicolas Maduro (d) Le Chang Lu (a) Turkey
(b) The United States

©LegalEdge Tutorials Page 41 of 60 ©LegalEdge Tutorials Page 42 of 60


Replication or other unauthorized use of this material is prohibited by the copyright laws of India Replication or other unauthorized use of this material is prohibited by the copyright laws of India
LE Prep Assist LE Prep Assist
(c) Austria 148. U.S. President Donald J. Trump signed into law 156. The Union Human Resource Development 165. The National Green Tribunal has imposed a fine of
(d) Qatar the ARIA. The act establishes a multifaceted U.S. Ministry constituted a task force to look into the 100 crore rupees on ________government for its
strategy to increase U.S. security, economic issues of suicides of the students in Jawahar failure to curb illegal coal mining in the state.
141. The MSME ministry has established an export interests, and values in the Indo-Pacific region. Navodaya Vidyalayas. The chairman of this (a) Meghalaya (b) Manipur
promotion cell to create a sustainable ecosystem What does I stands for in ARIA? committee is __________. (c) Jharkhand (d) Karnataka
for micro, small and medium enterprises. Who (a) Indo (b) India (a) Jitendra Nagpal (b) Jitendra Singh
is the present Minister of MSME? (c) International (d) Initiative (c) P K Sinha (d) Piyush Goyal 166. The 106th session of Indian Science Congress (IS(c),
(a) Giriraj Singh 2019 concluded in ________.
(b) Narendra Singh Tomar 149. The 'ARIA' will authorize ________ in spending 157. Who has become the first player to win three (a) Pune (b) Jalandhar
(c) Kalraj Mishra for a range of U.S. programs in East and Southeast Hopman Cups? (c) Lucknow (d) Dehradun
(d) Maneka Gandhi Asia. (a) Rafael Nadal (b) Novak Djokovic
(a) $1. Billion (b) $10 billion (c) Belinda Bencic (d) Roger Federer 167. A ministry Introduced 70-Point Grading Index. It is
142. Recently, Prime Minister Erna Solberg visited (c) $1.5 billion (d) $2.5 billion aimed to ______
India.She is the current PM of which country? 158. Ramnath Goenka Excellence awards is given in the (a) Assess the quality of Mid-day meal at
(a) UK (b) Denmark 150. Who among the following has become world s field of _________. government schools
(c) Australia (d) Norway first woman amputee to climb Mount Vinson, the (a) National Defence (b) Sports (b) Decrease the number of suicides in JNVs
highest peak in Antarctica? (c) Journalism (d) Social activism (c) Assess the quality of teachers' training
143. Which country became the first Asian team to (a) Santosh Yadav (b) Alana Nichols program in schools
win a Test series in Australia? (c) Melissa Stockwell (d) Arunima Sinha 159. The Union Cabinet approved a ______ reservation (d) Assess the quality of school education
(a) Bangladesh for economicall backward upper castes in
(b) Sri Lanka 151. The Indian Parliament passed the National government jobs. 168. According to the Central Statistics Office (CSO),
(c) Pakistan Council for Teacher Education (Amendment) Bill, (a) 8% (b) 12% income earned by an average Indian per year
(d) India 2018, following the approval of the Rajya Sabha. (c) 17% (d) 10% doubled in 7 years. What is the income earned by
This bill will replace the _________________. an average Indian in 2018-19?
144. Who won men s Hopman Cup 2018? (a) National Council for Teacher Education 160. The government has announced reservation in jobs (a) Rs 1.45 lakh (b) Rs 96,500
(a) Rafael Nadal (b) Alexander Zverev (NCTE) Act, 2003 for those belonging to the upper castes and earning (c) Rs 1.01 lakh (d) Rs 1.25 lakh
(c) Rohan Bopanna (d) Roger Federer (b) National Council for Teacher Education less than ______ per year.
(NCTE) Act, 1993 (a) Rs 8 lakh (b) Rs 5 lakh 169. Name the Malaysian King who has abdicated after
145. Who will head the nine-member high-level (c) Right of Children to Free and Compulsory (c) Rs 3 lakh (d) Rs 6 lakh two years on the throne, the first time a monarch
committee constituted by the Union Education Bill, 2009 has stepped down before completing their five-
government to look into the implementation of (d) National Council for Teacher's 161. Recently,Centre gives nod for centre for year tenure.
Clause 6 of the Assam accord of 1985? Training(NCTT) Act, 1999 classical language in which state? (a) Abdul Halim V
(a) Vikram Misri (a) Himachal Pradesh (b) Odisha (b) M. Muhammad III
(b) Vijay Keshav Gokhale 152. The International Kite Festival has been started (c) Kerala (d) TamilNadu (c) Muhammad V
(c) Pravveen Kumar in__________. (d) Tengku Muhammad Faiz Petra
(d) M.P. Bezbaruah (a) Vadodara (b) Ahmedabad 162. What is the name of the renminbi-denominated
(c) Gandhi Nagar (d) Surat bonds which has been approved b Pakistan s 170. What is the name of World Bank President who has
146. The Cabinet has approved the restructuring of cabinet to raise loans from China s capital resigned from the post?
National Health Agency as _______for better 153. Who is the Head of the High-Level Committee for markets? (a) Mariya Gabriel (b) Gita Gopinath
implementation of Pradhan Mantri Jan Arogya implementation of Clause VI of Assam Accord? (a) Dragon Bonds (c) António Guterres (d) Jim Yong Kim
Yojana. (a) MP Bezbarauah (b) T.N. Rao (b) China-Connect Bonds
(a) Rastriya AYUSH Mission (c) Venkatesh Pillai (d) Lovish Pal (c) Masala Bonds 171. Who has become the first woman chief economist
(b) National Health Authority (d) Panda Bonds of International Monetary Fund?
(c) National AYUSH Agency 154. In the United States, Department of Defence, Chief (a) Kristalina Georgieva
(d) Health Authority of India of Staff, Rear Admiral _______ has resigned. 163. Mrinal Sen passed away at the age of 95 in (b) Gita Gopinath
(a) Francis Sweeney (b) Joseph Dunford Kolkata. He was a well-known _________. (c) Jim Yong Kim
147. The Indian Parliament passed the Right of (c) Kevin Mattis (d) Kevin Sweeney (a) Filmmaker (b) Journalist (d) Geeta Gopikrishnan
Children to Free and Compulsory Education (c) Economist (d) Environmentalist
(Amendment) Bill, 2018. This amendment is 155. Name the Prime Minister of Norway who has 172. Who among the following has been appointed as
related to which of the following policy in the Act? arrived in New Delhi on a three-day state visit to 164. Prime Minister of Norway has arrived in New new Chief Information Commissioner (CI(c)?
(a) Teacher's training India. Delhi on a three-day state visit to India. Name the (a) BVP Rao
(b) No-detention (a) Erna Solberg (b) Angela Merkel Capital of Norway. (b) Sudhir Bhargava
(c) Zero tolerance (c) Theresa May (d) Jean-Claude Juncker (a) Basel (b) Alta (c) Harsh Vardhan Shringla
(d) Mid-day meal (c) Oslo (d) Molde (d) Suresh Chandra

©LegalEdge Tutorials Page 43 of 60 ©LegalEdge Tutorials Page 44 of 60


Replication or other unauthorized use of this material is prohibited by the copyright laws of India Replication or other unauthorized use of this material is prohibited by the copyright laws of India
LE Prep Assist LE Prep Assist
173. Which of the following Indian Cricketer has been community. What is the theme of the 2019 Pravasi 195. The Constitution Day has been observed across
suspended by the International Cricket Council 183. Exercise Konkan 18 is the annual bilateral Bhartiya Divas (15th Edition)? India on ____.
(IC(c)from bowling in International Cricket? maritime exercise between Indian Navy and the (a) Apna Bharat Apna Gaurav : Connecting (a) November 25 (b) November 26
(a) Ravindra Jadeja (b) Ambati Rayudu Royal Navy was held in which state? Across Generation (c) December 1 (d) December 2
(c) Yuvraj Singh (d) Suresh Raina (a) Maharashtra (b) West Bengal (b) Redefining Engagement with the Indian
(c) Goa (d) Tamil Nadu Diaspora 196. Dampa tiger reserve is located in which state?
174. The Anti-Lepros Da is being observed in India (c) Engaging Diaspora: Connecting Across (a) Meghalaya (b) Mizoram
on which day? 184. Who has won the Indonesia Masters women s Generations (c) Tripura (d) Assam
(a) January 28 (b) January 30 singles title 2019? (d) Role of Indian Diaspora in building New India
(c) February 2 (d) February 5 (a) Carolina Marin 197. Which of the following is the capital of
(b) P V Sindhu 192. Who has been appointed as the 26th Chief Justice U bekistan ?
175. Which of the following State government will (c) Saina Nehwal of Pakistan? (a) Astana (b) Ashgabat
form 'State Tiger Protection Force' (STPF) to save (d) Ratchanok Intanon (a) Justice Munib Akhtar (c) Dushanbe (d) Tashkent
the tiger population in the State? (b) Justice Ijazul Ahsan
(a) Bihar (b) Uttar Pradesh 185. Who was the Chief Guest at Republic Day (c) Justice Qazi Faez Isa 198. Which of the following is the capital of Ethiopia?
(c) Punjab (d) Telangana celebrations 2019 in New Delhi? (d) Justice Asif Saeed Khosa (a) Yerevan (b) Minsk
(a) François Hollande (c) Phnom Penh (d) Addis Ababa
176. Who has won the best Tableau Trophy of Republic (b) Shinzo Abe 193. Prime Minister Shri Narendra Modi has
Day Parade 2019 held at Rajpath,New Delhi? (c) Cyril Ramaphosa inaugurated the countr s first National Museum 199. What is the rank of the India in the Global Talent
(a)Jammu and Kashmir tableau (d) Donald Trump of Indian Cinema (NMI(c) in ______. Competitive Index (GTCI) 2019?
(b) Arunachal Pradesh tableau (a) Mumbai (b) Chennai (a) 81 (b) 59
(c) Uttarakhand tableau 186. Which city hosted Pravasi Bharatiya Divas in (c) Kolkata (d) New Delhi (c) 80 (d) 73
(d) Central Industrial Security Force (CISF) January 2019?
tableau (a) Varanasi (b) Kolkata 194. The Armed Forces Flag Day is observed annually 200. On which date Himachal Pradesh celebrated its
(c) Chennai (d) Bengaluru in India on which of the following Day? 49th Statehood Day?
177. Who has won the women s singles 2019 Australian (a) December 2 (b) December 4 (a) 24 January 2019 (b) 25 January 2019
Open Tennis Tournament? 187. The Pulitzer Prize-winning writer Russell Baker (c) December 5 (d) December 7 (c) 26 January 2019 (d) 22 January 2019
(a) Angelique Kerber (b) Simona Halep passed away at his home. He belonged to _____.
(c) Petra Kvitova (d) Naomi Osaka (a) Australia (b) Brazil
(c) France (d) USA
178. Who has won the Indonesia Masters men s singles
title 2019? 188. Which of the following former President of India
(a) Chen Long (b) Lee Chong Wei has been conferred upon Bharat Ratna for the
(c) Lin Dan (d) Anders Antonsen year 2019?
(a) Gyani Zail Singh
179. The 'Beating Retreat' ceremony held every year (b) Pratibha Devisingh Patil
on______. (c) R Venkataraman
(a) 30 th January (b) 15th January (d) Pranab Mukherjee
(c) 29 th January (d) 10th January
189. Who among the following has been awarded
180. Which of the following state Government will Bharat Ratna posthumously in 2019?
develop India s second Tulip garden? (a) Nanaji Deshmukh
(a) Assam (b) Bihar (b) Bhupen Hazarika
(c) Gujarat (d) Uttarakhand (c) Both A and B
(d) Gyani Zail Singh
181. Which of the following states Government has
launched One Famil One Job scheme? 190. Which of the following three states celebrated Full
(a) Punjab (b) Haryana Statehood Day on 21st January?
(c) Sikkim (d) Assam (a) Assam, West Bengal,and Odisha
(b) Punjab, Uttarakhand and Haryana
182. As part of the Republic Day 2019 celebrations, (c) Bihar, West Bengal and Uttar Pradesh
which of the following cit is hosting the Bharat (d) Manipur, Meghalaya and Tripura
Parv festival?
(a) Mumbai (b) Pune 191. Pravasi Bhartiya Divas is celebrated every year to
(c) Chandigarh (d) Delhi mark the contribution of the overseas Indian
©LegalEdge Tutorials Page 45 of 60 ©LegalEdge Tutorials Page 46 of 60
Replication or other unauthorized use of this material is prohibited by the copyright laws of India Replication or other unauthorized use of this material is prohibited by the copyright laws of India
LE Prep Assist LE Prep Assist
17. As per the principle, a person who was involved in a 33. Nowhere it is suggested in the facts that X made a
wrongdoing at the time of the alleged negligence will statement with respect to his attendance thinking the
ANSWER KEY AND EXPLANATIONS
Prep Test Legal Aptitude face the illegalit defence whereb the defendant s statement to be false. He maintained his silence all
liability will be reduced or extinguished. In the instant throughout.
1. b 2. b 3. d 4. c 5. b 6. b 7. d 8. a 9. b 10. d case, Lawrence had illegall entered Rupert s house and 34. The principle clearly states that mere silence on the issue
11. d 12. c 13. c 14. a 15. c 16. a 17. a 18. d 19. b 20. d had broken his music s stem, when Rupert s voice will not constitute fraud. As X stayed silent on the entire
21. a 22. c 23. a 24. b 25. a 26. d 27. c 28. b 29. a 30. c startled him and he burnt himself. Thus, statements I and matter, it will not constitute fraud.
31. b 32. c 33. a 34. b 35. c 36. a 37. a 38. c 39. b 40. c III hold true in this case. 35. As no statement was made, there can be no
18. The loss or damage suffered by Faraz cannot be said to intention to deceive. However, X s silence
41. d 42. b 43. c 44. b 45. a 46. d 47. a 48. d 49. b 50. d
include the price that he was ready to pay for the Camera
misrepresented the facts in a different light.
to Karn; it is limited to the extra money he will now have
1. Literal meaning of Sovereignty is "state of making laws However, there is no maximum age limit prescribed for 36. Clearly, all the elements necessary to constitute 'theft' as
to pay in light of the breach of the contract. Moreover,
or liberty to decide one's thoughts and actions". a person to be elected as President of India. defined in the principle have been satisfied. Since
considerations such as inflation are not damages that
Parliament sovereignty is a concept in some parliament 10. Article 371-A deals with special provision with respect to Bubbles took away Buttercup's bracelet from her
arise in the natural consequence of the breach.
democracies; it holds that Legislative body has absolute the State of Nagaland. This Article accorded the special ownership, for an illegal purpose, she is guilty of theft.
19. The damage caused is too remote and indirect and the
sovereignty and has discretionary power in making laws. power in respect of religious and social practices of 37. The principle clearly refers to use of threat or force. In the
'but for' standard is not optimum standard since it can
Best example of Parliamentary form of Government is Nagas, Nagas customary law and procedure and Special present case, the State of Gulan merely provided military
lead to such remote damages being attributed to Faraz's
the United Kingdom. responsibility of Governor with respect to law and order aid to suppress the uprising, and that too, at the request
breach.
2. Right to Equality covers Article 14 Article 18 of Part III. in the State and many more other provisions. of the officials of the State of Tulli.
20. To say that Kanta will have to enter the agreement
Article 15 states that State shall not discriminate against 11. The National Commission to Review the Working of the 38. As the first principle states, the general rule is that the
without free consent is to beg the question. The very
any citizen on grounds only of religion, race, caste, sex, Constitution was set up by Government dated 22 right to sue or recover damages is not transferable.
question here is whether there is coercion; which if it
place of birth or any of them. In context of this question, February, 2000 to examine, the experience of the past 50 However, principle II states the exception to the rule,
exists, will vitiate the free consent. Residence of parties in
public office comes in the ambit of state and thus by years, as to how best the Constitution can respond to the which is that the right to recover damages may be
the state of Jammu and Kashmir is, as explained,
denying any person access to public office because of his changing needs of efficient, smooth and effective system transferred if it is accompanied with a transfer of
immaterial.
religion, state is violating his Right to Equality. of governance, and to recommend changes, without property.
21. It is clear from the principle that the fight must be taking
3. As per Article 84 of the Indian Constitution, the age for changing the basic structure. Former Chief Justice of 39. The last part of Principle II clearly mentions that an
place in a public place; a terrace is not a public place no
contesting election for the Lok Sabha is 25 years. India Justice M.N Venkatachaliah was the Chairperson amendment to the objects clause, with regard to a new
matter how close to one it may be.
4. Rajya Sabha is also known as Council of States or Upper of the commission. business, is subject to certain conditions. The business of
22. The accident in this case was avoidable as Neel knew that
House or House of Elders. Article 80 deals with 12. Article 330 and Article 332 of the Constitution of India manufacturing pesticides is in total contradiction of
the roads were slippery. His care and caution was only
composition of Rajya Sabha. The composition of Rajya deals with the Reservation of seats for the Scheduled organic farming that employs the use of natural products
aimed at minimizing the likelihood of the potential slip
Sabha is as follows: Caste and Scheduled Tribes in the House of the People and uses no chemical fertilizers or pesticides.
and not at avoiding it. Whether or not hundreds of lives
I. There are total 250 members out of which 238 and in the Legislative Assemblies of the state 40. The principles clearly outline the articles of the
were lost due to Neel's negligence is not decided at this
members are elected from States and UTs while 12 respectively. Of the total strength of 543, in Lok Sabha, 84 Constitution pertaining to individual responsibility
stage and is therefore irrelevant.
members are nominated by the President of India. seats are served for the Scheduled Castes and 47 for the towards the environment. Breach of the same can be
23. (The principle of vicarious liabilit shall appl as Z s
II. The term of a Rajya Sabha member is six years and the Scheduled Tribes. All the other options are not present in considered to be a reasonable restriction on the freedom
negligence was committed during the course of his
same shall not be subject to dissolution that is why the Constitution. to carry on trade and business.
employment under Y)
1/3rd of its members thereof shall retire every alternate 13. As per the principle and accompanying explanation, 41. The agreement merely imposes a waiting period of three
24. (The principle of vicarious liability shall apply, holding
year. Tinu had direct intent when he injured Samrat while months, but it does not completely oust the jurisdiction
X liable for the tort of his employee - Y.)
5. Article 159 defines the oath to the Governor of state. transferred intent was involved when Kamran got of the courts. Hence, it is valid.
25. (The Committee showed negligent conduct by not
According to this Article, Oath of the Governor is injured. Samrat clearly suffered from a reasonably 42. It ma have been the electrician s carelessness, but being
taking proper care of the buildings of the town and
administered by Chief Justice of High Court exercising foreseeable harm. the owner, it was his responsibility to ensure that no one
putting the safety of the residents at risk. The defence of
jurisdiction in relation to that state or in his absence 14. As per the principle and the explanation, it is clear that was injured in his premises. The ultimate responsibility
lack of funds would not absolve them from liability)
senior most judge of that High Court administers the transferred intent comes to the fore when a person is his.
26. is clearly bound by the acceptance as it was put in
same. Similarly, the oath of the President is administered intends to injure an individual but ends up injuring 43. This is a valid case of exercising the right of self defence.
transmission before 30 th January)
by the Chief Justice of India. someone else. In the instant case, Tinu intended to hurt In the given circumstances Ram had received nine
27. (Self-explanatory)
6. The Election to the office of President is conducted by Samrat but ended up injuring another person. Hence, injuries on his body and he would not have survived if
28. There was clearly an active concealment of facts and
Election Commission of India. Election and Manner of transferred intent is involved. he had received any more bullet injuries. Therefore, Ram
therefore the contract is voidable at the option of X.)
Election of President is defined under Article 54 and 15. Principle (i) and the first head under principle (ii) clarify can validly claim right of self defence.
29. (Self-explanatory)
Article 55 of the Constitution of India respectively. that trespass to chattel is an intentional interference with 44. The government can choose a tender other than the
30. Sukanya was talking on the phone while walking back
7. In 1776, Sweden was the first country to enact Right to the possession of personal property, resulting in damage lowest one only if justifiable reasons are given. Here the
home at night. It does not imply her negligence in any
Information Law. It largel motivated the Parliament s and the fact that there must be a lack of consent in such action was arbitrary.
manner whatsoever.
interest in access to the information held by king. interference. The facts in the instant case show that 45. In option (a), master-servant relationship can be
31. The facts clearly mention that Vijay was in a state of
8. The Union of India has no power to declare Financial Sabrina had given implied consent for Edward to go and established if the team of employees are hired by the
intoxication and was not quite aware of his
Emergency under Article 360 in the state of Jammu and get the clock oiled etc. Thus, she may not sue him for owner of the apartment. Hence, the owner can be held
surroundings. Thus, while driving in this condition, he
Kashmir. The Union can declare State Emergency under trespass to chattel. liable vicariously.
could have caused an accident. Hence, Sukan a s death
Article 356 in the state only in case of War or External 16. Principles (i) and (ii) deal with the essential ingredients 46. Here A genuinely believed in good faith that Z is Y. This
could have been reasonably foreseeable due to Vija s
Aggression. No proclamation of emergency made on the to constitute trespass and it may be seen that in the is a mistake of fact and hence A has not committed any
condition and his actions.
grounds of internal disturbance or imminent danger. instant case, Wilson intentionally interfered with offence.
32. Explanation: The facts clearl mention that Vija s boss
9. Article 58 of Constitution of India Qualification for Wilbur s logbook, without his consent and actual harm 47. It was default by Chingiz which led to the escape and
insisted on him drinking the alcohol. However, Vijay
election as President provides that the minimum age resulted from it. Thus, Wilbur may sue Wilson for consequent damage.
could anytime refuse to the same. He was not being force
for a person to be elected as President is 35 years. trespass to chattel. 48. N.A.
fed. Hence, there was no involuntary intoxication.

©LegalEdge Tutorials Page 47 of 60 ©LegalEdge Tutorials Page 48 of 60


Replication or other unauthorized use of this material is prohibited by the copyright laws of India Replication or other unauthorized use of this material is prohibited by the copyright laws of India
LE Prep Assist LE Prep Assist
49. Consolidated fund is the chief account of the educated members of OBCs should not benefit from the SP =x+ Rs.1.2 On solving x=1600
Government of India. No amountss can be withdrawn government sponsored schemes and programmes. The Profit = 40% Selling Price = 1600 × 125/100= Rs. 2000
from the fund without the authorisation from the Mandal Commission in 1979 recommended a 27% Therefore, we can say 16. Ans: (b) Explanation: Cost price of 1 Toy = 375/12 =
parliament. Fund is formed under the provision of reservation for OBCs in government jobs. The criteria to 1.40x = x+1.2 or x = Rs. 3 31.25
Article 266(1) of Indian Constitution and provision of determine creamy layer is currently Rs. 6,00,000 per Hence, Original SP = Rs 1.2x = Rs. 3.60/- Selling price of 1 toy = Rs.33
Article 266(3) states that no money can be appropriated annum. However, the National Commission for 10. Ans: (c) Explanation: Profit= 5% (If case) Profit= Rs (33 - 31.25) =1.75
from consolidated funds except in accordance with law. Backward Classes in October, 2015, has proposed to raise 5% of CP ------> Rs. 27 Profit% = 1.75/31.25×100= 5.6
50. The test of creamy layer in India is only applicable to the minimum celling to Rs. 15,00,000 of annual family So, CP = Rs. 540 17. Ans: (c) Explanation: Lcm of 5&6 =30
OBCs. The term means that the relatively wealthier and income. Cost price of 30 articles= 5/6 × 30 = 25
Now, Loss% = 10
Selling Price of 30 articles = 6/5 × 30 =36
Loss =Rs. 54
% profit = (36-25)/25 × 100= 44%
Required % = 54×100/27 = 200% 18. Ans: (b) Explanation: Cost price of 17 toys Selling
11. Ans: (c) Explanation: Part sold at 24% profit = 1-(1/3 + price of 17 toys = cost Price of 5 toys
1/4) = 5/12
ANSWER KEY AND EXPLANATIONS
Cost price of 12 toys = selling Price of 17 toys= 720
Value of commodity = (80×100) / (1/3 × 15 + 1/4 × Cost price of 1 toy= 720/12= 60
Prep Test Mathematics
20+5/12 × 24)= 400 19. Ans: (c) Explanation: Both I & II are sufficient to find the
1. A 2. B 3. C 4. D 5. A 6. B 7. A 8. D 9. D 10. C 12. Ans: (b) Explanation: Gain or loss = [2×5×4/4(5 + 4) - 1] exact amount of profit.
11. C 12. B 13. D 14. A 15. D 16. B 17. C 18. B 19. C 20. C × 100 % = 11.11% 20. Ans: (c) Explanation: Selling Price With Profit of 10 %
Sign is + ive so gain 11.11% of total cost = 3000×10 × 110/100 = 33000
1. Ans: (a)Explanation: Total Number of printers = 5 (2 5. Ans: (a) Explanation: Let SP be Rs. 100 13. Ans: (d) Explanation: 10 = x-5 5x/100 Selling Price of 2 cows with 5% loss= 6000 × 95/100 =
sold, 3 unsold) CP for Sunil = => (SP-CP)×100/SP = 50 19x/20 = 15; x=15.789% = approx 15.8% 5700
CP for Sunil ==> (100-CP)×100/100 = 50 or CP = Rs. 50 14. Ans: (a) Explanation: Let C.P. be Rs. x and S.P. be Rs. y. Difference = 33000 - 5700= 27300;
Monitors = 20.
(Divided by SP as Profit calculated on SP) Then, 3(y - x) = (2y - x) y = 2x. So rate of the cows for selling to gain 10% profit on total
Profit made on Printers sold = 2000×2 = 4000. Profit = Rs. (y - x) = Rs. (2x - x) = Rs. x.
Profit for Sunil = 100-50 = Rs 50 = 27300/7 = Rs. 3900
Monitors sold = 20 75% = 15 Now, CP for Sujeet = (SP-CP)×100/CP = 40 So profit % =100% (because 1 cow died so remaining= 10 1 2 = 7 )
Profit made on Monitors sold = 49000-4000 = Rs.45000. (100-CP)×100= 40CP or CP for Sujeet= Rs. 1000/14 15. Ans: (d) Explanation: Let CP is x.
Profit made per monitor = 45000/15 = 3000. Profit for Sujeet = 100-100/14 = 400/14 Then (1920 - x)/x × 100= (x - 1280)/x ×100
20% of CP of Monitor = 3000 Now, Difference in profit when SP is 100 = 50-400/14 =
CP of Monitor = 15000. 300/14.
CP of Printer = 7500 Now, Equating difference and SP, we have
300/14 : 100 : : 900 : SP
Total CP = 15000 20 + 7500×5 = 3, 37,500
SP = 900×100×14/300 = Rs. 4200
Total SP = 18000 15 + 9500×2 = 2, 89,000 6. Ans: (b) Explanation: Let originally he buy X kg for Rs.
Loss = 48,500 180
2. Ans: (b)Explanation: Total investment = Rs. 4725 Now, he will buy X+2 kg for Rs. 180.
Total SP = 1.4 4725 = 6615 Reduction in original price =10%
Now, Let the price of 4 seater be x then price of two (180/X)/kg × 90/100 = [180/(X+2)]/kg
90(X+2) = 100X
seater will be .75x.
X = 18
8x + 22 0.75x = 6615
Therefore, Originally he bought 18kg.
24.5x= 6615 or x = 270 Original Price = Rs. 10/kg
3. Ans: (c)Explanation: Total number of Microwave ovens Reduced Price = Rs. 9/kg
= 15 (12 sold +3 unsold) 7. Ans: (a) Explanation: SP = Rs. 24
Hence, Washing machines = 10 Let CP be X hence, Loss% = X
He sold 12 ovens and 8 washing machines (X-SP)× 100/X = X or (X-24) × 100/X = X
X^2-100X + 2400 = 0
Hence, In total he sold 80% of both
(X-60) (X-40) = 0
Thus, He sells 80% of both at a profit of Rs. 40,000.
X= 60 or 40
Cost of 80% of the goods = 0.8 2,05,000 = 1,64,000 8. Ans: (d) Explanation: Let CP of one be X and other be Y
Hence, Total SP = 1,64,000+40,000 = 2,04,000 X+Y = 1550 .(i)
CP = 2,05,000 ATQ:-
Loss = Rs.1000 0.77X + 1.27Y = 1550 (ii) (as no profit and no loss is
4. Ans: (d) Explanation: After 2 years :- there)
Flat would be worth = 2Lakh× 1.2×1.2 = Rs. 288000 Solving both, we get
Land would be worth= 2.2Lakh×1.1×1.1 = Rs. 266200 50Y = 35650 or Y = 713
Profit of the Gainer = Rs. 21800 Hence, X = 1550-713 = 837
Profit % of the gainer = 21800×100/266200= Therefore, CP of each horse = 837,713
8.189(approx) 9. Ans: (d) Explanation: Let the CP be Rs. x/ orange
Also if loss% woudd have been asked of the loser Profit = 20%
loss% = 21800×100/288000 = 7.56 (approx. ) SP = Rs. 1.20x
Now, If case :-
©LegalEdge Tutorials Page 49 of 60 ©LegalEdge Tutorials Page 50 of 60
Replication or other unauthorized use of this material is prohibited by the copyright laws of India Replication or other unauthorized use of this material is prohibited by the copyright laws of India
LE Prep Assist LE Prep Assist
Everest in 2013, has now become the first women 34. The Appoinments Committee of the Cabinet has
1. C 2. b 3. c 4. b 5. c 6. d 7. d 8. c 9. d 10. b amputee to climb Mount Vinson(Highest peak of approved the appointment of VK Yadav as railway
11. b 12. d 13. b 14. a 15. b 16. c 17. d 18. d 19. b 20. a Antarctica. board Chairman on the superannuation of Ashwani
21. c 22. d 23. d 24. a 25. c 26. d 27. b 28. a 29. b 30. d 21. World Braille Day is annually celebrated on Jan 4, the Lohani on December 31
31. d 32. c 33. b 34. a 35. b 36. d 37. a 38. b 39. c 40. d birthday of Braille inventor, Louis Braille. 35. The government has appointed Sudhir Bhargava as
41. c 42. b 43. d 44. d 45. a 46. a 47. d 48. b 49. a 50. c 22. The Reserve Bank of India (RBI) constituted an expert the new Chief Information Commissioner.
51. d 52. b 53. c 54. b 55. d 56. d 57. d 58. c 59. d 60. a committee, which will review the current 36. Justice G Rohini(ret(d), who is heading the five
institutional framework in place to support MSMEs member panel, set up to suggest parameters for an
61. d 62. d 63. c 64. a 65. c 66. d 67. a 68. a 69. d 70. d
along with examining the factors affecting timely and equitable distribution of OBC reservation.
71. a 72. b 73. b 74. b 75. b 76. d 77. d 78. c 79. a 80. c
adequate availabilit of finance to them. The new 8- 37. Microsoft has selected Kerala Police s Facebook Page,
81. d 82. b 83. c 84. c 85. d 86. d 87. d 88. b 89. d 90. d
member committee would be chaired by Upendra which has over 10 lakh followers, to study the
91. c 92. b 93. a 94. c 95. a 96. a 97. a 98. c 99. b 100. b
Kumar Sinha. organisation s new age media interactions.
101. a 102. a 103. a 104. c 105. b 106. d 107. b 108. a 109. b 110. c 23. Veteran cricket coach Ramakant Achrekar, who 38. Explanation: A Chinese space probe is moving into
111. c 112. c 113. d 114. b 115. d 116. a 117. b 118. a 119. d 120. d trained Bharat Ratna Sachin Tendulkar and his friend position to land on the dark side of the moon for the
121. c 122. a 123. b 124. d 125. c 126. d 127. d 128. c 129. c 130. d Vinod Kambli, among others, passed away following first time. The probe,the Chang e-4, entered a planned
131. d 132. c 133. d 134. d 135. c 136. d 137. d 138. b 139. c 140. d a heart attack. orbit to prepare for the first ever soft landing on the
141. a 142. d 143. d 144. d 145. d 146. b 147. b 148. d 149. c 150. d 24. The government has constituted a committee to far side of the moon .
151. b 152. b 153. a 154. d 155. a 156. a 157. d 158. c 159. d 160. a suggest recommendations to strengthen the real estate 39. Uzbekistan Shavkat Mirziyoyev in his annual address
161. c 162. d 163. a 164. c 165. a 166. b 167. d 168. d 169. c 170. d law RERA and to remove difficulties in its to the Parliament announced 2019 as the Year of
171. b 172. b 173. b 174. b 175. d 176. d 177. d 178. d 179. c 180. d implementation.The decision to form the committee, Active Investment and Social Development in the
181. c 182. d 183. c 184. c 185. c 186. a 187. d 188. d 189. c 190. d to be headed by Union Housing and Urban Affairs backdrop of proposed investments from India to boost
191. d 192. d 193. a 194. d 195. b 196. b 197. d 198. d 199. c 200. b Joint Secretary Shiv Das Meena. economy of this Central Asian country.
25. In a wonderful initiative to support gender equality 40. Sultan Abdullah Sultan Ahmad Shah was elected as
and renaissance value, around 50 lakh women formed Mala sia s new king b the members of the countr s
a 620 km long Women s Wall in Kerala. royal families. He replaced Sultan Muhammad V of
1. ISRO chief K Sivan said the Rs 10,000 crore worth 10. Andhra Pradesh has topped the Ease of Doing Business
26. As part of the enhanced outreach programme of Kelantan, who stepped down recently, making it the first
Gaganyaan mission is a major turning point for the rankings. Maharashtra and Delhi came in second and
Indian Space Research Organisation (ISRO), a new abdication in Mala sia s histor .
space agency, while adding that his team is targeting third place respectively.
platform named Samwad with Students was launched 41. Eminent Hindi author and essayist Krishna Sobti passed
to send the first manned mission to space in December 11. Malaysia's 49-year-old Sultan Muhammad V stepped
in Bengaluru on January 1, 2019. away in Delhi. She was 93. She was a recipient of many
2021. down as the 15th king on Sunday, marking the first time
27. The United States and Israel officiall quit the U.N. s coveted awards like Sahitya Akademi, Jnanpith awards
2. Jair Bolsonaro was sworn in as Bra il s President in a a king has abdicated in the country since it gained
educational, scientific and cultural agency (UNESCO) and was also offered Padma Bhushan, which she had
ceremon at Bra il s National Congress Building. independence from Britain in 1957.
at the stroke of midnight, the culmination of a process declined.
Bolsonaro is a former Army captain and admirer of the 12. Krishnamurth Subramanian is India s Chief Economic
triggered more than a year ago amid concerns that the 42. The Defence Research and Development Organisation
countr s 1964-1985 military dictatorship. Advisor.
organization fosters anti-Israel bias. (DRDO) has successfully test fired Long Range Surface to
3. The government launched Web Wonder Women 13. New Delhi: Justices Dinesh Maheshwari and Sanjiv
28. France will introduce its own tax on large internet and Air Missile (LR-SAM) from naval warship INS Chennai.
Campaign to celebrate the exceptional achievements Khanna were sworn in as Supreme Court judges by
technology companies from January 1. It has been The test was conducted off the coast of Odisha.
of women who have influenced and impacted society Chief Justice of India Ranjan Gogoi. Maheshwari was
pushing hard for a new so-called GAFA tax-named 43. This year, 8th Battalion of National Disaster Response
through social media. sworn in first, followed by Khanna.
after Google, Apple, Facebook and Ama on to Force located at Ghaziabad has been selected for the
4. A five-Judge Constitution bench of the Supreme Court 14. The Insolvency and Bankruptcy Board of India (IBBI)
ensure the global giants pay a fair share of taxes on Award for its commendable work in disaster
headed by CJI Ranjan Gogoi has been constituted to has set up a 11-member working group under the
their massive business operations in Europe. management.
hear the title suit in the Ram Janmabhoomi-Babri chairmanship of former SEBI Chief UK Sinha to go
29. Prime Minister Narendra Modi has inaugurated the 44. 25 January is celebrated every year as National Voters
Masjid Ayodhya case. into the concept of Group Insolvenc and suggest a
106th Indian Science Congress (ISC-2019) at Jalandhar, Day to spread awareness about the necessity of voting
5. Marking a golden letter day in the history of suitable framework.
Punjab. The theme for which is Future: India-Science and to encourage young generation towards the voting
Polavaram project-the life lie of Andhra Pradesh, the 15. Andry Rajoelina was sworn in as President of
and Technolog . rights.
concrete works of spill channel successfully entered Madagascar.
30. Veteran cricket coach Ramakant Achrekar, who trained 45. The all-women Assam Rifles contingent created history
Guinness Book of World Records by completing 16. The US City of Louisville in Kentucky state to rename
Bharat Ratna Sachin Tendulkar and Vinod Kambli, this year by participating for the first time in a Republic
32,315.5 Cubic meter of concrete works in a span of 24 its airport after hometown hero Muhammad Ali
among others, passed away following a heart attack. Day (26th January) parade. Contingents of the Navy,
hours. (name before conversion to Islam Cassius Clay), who
31. Veer Savarkar International Airport in Port Blair has India Army Service Corps and a unit of Corps of Signals
6. Even though the number of leprosy cases have died in 2016.
been declared as an authorized Immigration were all led by women officers.
steadily declined worldwide, an estimated two lakh 17. Vijay Mallya became the first tycoon to be charged
Check post for entry into/exit from India with valid 46. India s fastest indigenous train which will run from Delhi
cases continue to be reported every year, with India under a new anti fraud law, with a court in Mumbai
travel documents for all classes of passengers. to Varanasi has been named as Vande Bharat Express. Its
accounting for more than half,said WHO. naming him as an offender under the Fugitive
32. Justice Thottathil Bhaskaran Nair Radhakrishnan was previous name was Train 18. The new name has been
7. The world s longest 3D-printed concrete pedestrian Economic Offenders Act, 2018.
sworn in as the first Chief Justice of the Telangana High given after taking suggestions from the general public.
bridge, which is 26.3 metre-long and 3.6-metre-wide, 18. Thousands have been left stranded across tourist
Court. The oath to the office was administered to Justice 47. Novak Djokovic won the Australian Open 2019, Men s
opened in China s Shanghai. hotspots in Thailand as tropical storm Pabuk made
Radhakrishnan by Telangana and Andhra Pradesh Singles tennis title.
8. the Pakistan government declared sugarcane juice as landfall in the countr s southern gulf coast.
Governor ESL Narasimhan at a ceremony held at Raj 48. Naomi Osaka won the Australian Open 2019, Women s
the national drink of the countr . 19. U.S. President Donald J Trump signed into law the
Bhavan. Singles tennis title.
9. Roger Federer has won the Hopman Cup for a record Asia Reassurance Initiative Act, which passed the U.S.
33. Tropical Storm Usman, which entered the Philippines 49. Former Defence Minister George Fernandes passed away
third time after Switzerland beat Germany in a winner- Senate earlier in December.
on Christmas Day, emerged as the deadliest weather in Delhi at the age of 88. Fernandes also served as the
takes-all point. 20. Arunima Sinha, the Indian mountaineer who became
disaster for the country this year following Typhoon Railwa s Minister in former Prime Minister VP Singh s
the world s first woman amputee to climb Mount
Mangkhut in September.

©LegalEdge Tutorials Page 51 of 60 ©LegalEdge Tutorials Page 52 of 60


Replication or other unauthorized use of this material is prohibited by the copyright laws of India Replication or other unauthorized use of this material is prohibited by the copyright laws of India
LE Prep Assist LE Prep Assist
government. His last stint as a parliamentarian was as a 64. Himachal Pradesh MP Anurag Thakur has been tried in the infamous Baroda dynamite case. He was after her opponent Carolina Marin pulled out of the
Rajya Sabha MP between August 2009 and July 2010. bestowed with Sansad Ratna Award for his elected to the Lok Sabha nine times and once to the Rajya summit clash due to an injury.
50. A 5.1-km long cable-sta ed Atal Setu on the Mandovi distinguished performance as a parliamentarian under Sabha. 93. South Africa President Cyril Ramaphosa arrived in
river in Goa, a third such bridge to connect the state Jury Committee Special Award category. 79. The first National Consultation on Child Protection Delhi for his first state visit ahead of the Republic day
capital with North Goa, was opened to the public by 65. Solution: Nepal s central bank has banned the use of for 2019 was held at India Habitat Centre, New Delhi celebration parade, where he is the chief guest.
Union Minister Nitin Gadkari. Indian currency notes of Rs 2,000, Rs 500 and Rs 200 under the Chairmanship of the Secretary, Ministry of Ramaphosa is 2nd South Africa s President after
51. Suman Kumari has become the first Hindu woman to denominations, a move that could affect Indian tourists Women and Child Development. Nelson Mandela to be the chief guest at the Republic
have been appointed as a civil judge in Pakistan after visiting the Himalayan nation where Indian currency is 80. Egypt will host the 2019 Africa Cup of Nations Day parade.
passing an examination for induction of judicial officers. widely used. between 15th June and 13th July. The CAF executive 94. Explanation: The celebration of 15th Pravasi Bhartiya
Kumari, who hails from Qambar-Shahdadkot, will serve 66. Solution: Prime Minister of Mauritius Mr. Pravind committee preferred Egypt to South Africa as Divas was commenced in Varanasi,UP. Prime
in her native district. Jugnauth is the Chief Guest of the Pravasi Bharatiya replacement for original hosts Cameroon, who were Minister of Mauritius, Mr. Pravind Jugnauth was the
52. India has replaced Japan as the world s 2nd largest steel Diwas. convention: dropped due to delays in preparations and concerns chief Guest of Convention.
producing country, while China is the largest producer 67. Solution: India has moved up one position to rank 80th over security. 95. India opened the ear s space campaign b putting
of crude steel, accounting for more than 51% of on the global talent competitive index, but remains a 81. Nobel Peace Prize winner and Pakistani activist into orbit defence imaging satellite Microsoft R for
production, according to the World Steel Association. laggard among the BRICS nations. Switzerland continues Malala Yousaf ai pens a new book, titled we are the DRDO and students-built nano-satellite
The global steel body, in its latest report, stated that to top the list. displaced:My Journey and Stories from Refugee girls Kalamsat in a cop book st le.
China s crude steel output jumped 6.6% to 928.3 million 68. Solution: Andhra Pradesh cabinet has decided to cater around the world charting her experiences traveling 96. Australia s wicketkeeper-batter Alyssa Healy voted ICC
tonnes in 2018 from 870.9 MT in 2017. 5% reservation each for Kapu community and the the world and visiting refugee camps. Women s T20I Pla er of the Year.
53. India will host the 14th session of the Conference of economically backward classes (EBCs) among the 82. A 2+2 Inter-Sessional meeting between the United States 97. The three renamed islands of Andaman and Nicobar are:
Parties to the United Nations Convention to Combat forward castes. The cabinet approved the decision of and India took place in New Delhi, focusing on the 1: Ross renamed as Netaji Subhas Chandra Bose Dweep,
Desertification (UNCC(d) to address the issue of land doubling the welfare pensions given under the NTR progress of defence and foreign policy issues between the 2: Neil renamed as Shaheed Dweep, 3: Havelock
degradation and desertification which will have Bharosa Scheme. two countries. renamed as Swaraj Dweep.
participation from over 100 countries. It is likely to be 69. Solution: Ten ears after 26/11 , the Indian Nav 83. Sikkim Chief Minister Pawan Kumar Chamling launched 98. World Hindi day is celebrated on January 10 every
held in October 2019. commenced the largest coastal defence exercise off the the One Famil One Job scheme, under which for ever year, marking the anniversary of first World Hindi
54. Nashik in Maharashtra will be the site of the countr s Indian coast. Exercise SEA VIGIL, a first of its kind, is family one government job has been allotted, in Gangtok. Conference which was held in 1975.
second defence innovation hub after Coimbatore in being undertaken along the entire 7516.6 km coastline New recruits were taken for Group C and Group D posts 99. Acclaimed Hindi fiction writer and essayist Krishna
Tamil Nadu. and Exclusive Economic Zone of India and is involving in 12 departments. Sobti passed away in New Delhi at the age of 93 due
55. India has signed a long-term contract with Uzbekistan all the 13 coastal States and Union Territories along with 84. The National Youth Day is celebrated across the country to prolonged illness.
for supply of uranium ore concentrates to widen its all maritime stakeholders, including the fishing and on 12th January across the country. NYD 2019 is the 156th 100. The 2nd World Integrated Medicine Forum 2019 on
source base for the nuclear fuel. coastal communities. birth anniversary of Swami Vivekananda. the Regulation of Homeopathic Medical Products held
56. The Department of Personnel & Training, Government of 70. Solution: The World Economic Forum released the 14th 85. External Affairs Minister Sushma Swaraj embarked on a in Goa. AYUSH Minister Shripad Naik inaugurated
India has appointed Senior IPS Officer Prabhat Singh as edition of its annual Global Risk Report, 2019. The two-day visit to Uzbekistan. Uzbekistan Capital: the three day forum. 20 countries will participate in
Director General (Investigation) in the National Human report s findings are based on the annual Global Risks Tashkent, Currency: Uzbekistani s om. the event.
Rights Commission. Perception Survey, in which nearly 1,000 decision- 86. ISRO Chairman Dr. K Sivan has announced that the 101. Indians have emerged as the biggest supporters of
57. The Swedish Parliament approved Stefan Lofven as makers from the public sector, private sector, academia, organization is well prepared to meet the targets of international aid, with a global public opinion survey
Prime Minister for a second four-year term. and civil society assess the risks facing the world. launching Gaganyaan manned mission to outer space by putting India on the top when it comes to people
58. Solution: The United Nations Educational, Scientific, and 71. Solution: Fitch Rating company Headquarters is in New December 2021. Under Gaganyaan project, India is expecting their nation to help other countries.
Cultural Organization (UNESCO) has named the York. planning to send three astronauts to outer space for seven 102. Noted Gujarati poet Sitanshu Yashaschandra has been
Brazilian city of Rio de Janeiro as the World Capital of 72. A ministerial panel headed by Maharashtra Finance days and bring them back. awarded Saraswati Samman 2017 for his poetry
Architecture for 2020. The city beat Paris and Melbourne Minister Sudhir Mungantiwar will examine uniformity 87. Taiwan President Tsai Ing-wen has appointed Su Tseng- collection in Gujarati named Vakhar .
to get the position. of taxation on lottery under GST or other issues arising chang as Prime Minister, during a Cabinet reshuffle 103. Solution: National Voters Da Theme 2019 is No Voter
59. Solution: Defence Minister Nirmala Sitharaman out of it. following the ruling Democratic Progressive Part s to be left behind . This is the ninth ear of celebrating
inaugurated the 426 meters long Diffo Bridge over Chipu 73. The Gandhi Peace Prize for the year 2018 has been heavy losses in local elections. National Voters Day.
River in Lower Dibang Valley District of Arunachal conferred on Yohei Sasakawa. 88. Venezuela Capital- Caracas, Currency- Venezuelan 104. Kalamsat is a ten cm cube weighing one-kilogram
Pradesh. 74. Mohammed Ali Qamar, the man who won India s first bolívar. communication satellite with a life span of two months.
60. The 15th edition of Pravasi Bhartiya Divas begins at Commonwealth Games gold medal in boxing, has taken 89. Highlighting the success of Make in India and 105. The Telangana government has decided to form a State
Varanasi in Uttar Pradesh. The 3-day event is being over as the chief coach for the countr s female boxers. contribution of Indian workers, Minister of Railways Tiger Protection Force to save the Big Cat population in
organised in Varanasi for the first time. He has replaced veteran Shiv Singh. Pi ush Go al, renamed India s first indigenous semi- the State.
61. The theme this ear's Pravasi Bharti a Divas is, Role of 75. The government has constituted a seven-member Group high speed train Train 18 as Vande Bharat 106. Solution: National Voters Day was first celebrated in
Indian Diaspora in building New India . of Ministers (GoM) to boost the real estate sector under Express . 2011.
62. Solution: The Insolvency and Bankruptcy Board of India the GST regime. Deputy Chief Minister of Gujarat Nitin 90. GlaxoSmithKline Consumer HealthCare said its 107. Arunachal Pradesh Chief Minister, Pema Khandu
(IBBI) has set up an 11-member working group under the Patel is the convener of the GoM. merger deal with Hindustan Unilever Limited(HUL) declared Pakke Paga Hornbill Festival (PPHF) as the
Chairmanship of former SEBI chief UK Sinha to go into 76. Skopje is the capital of Macedonia. has been approved by the fair trade regulator CCI. State Festival at Seijosa in East Kameng district,
the concept of Group Insolvenc and suggest a suitable 77. Nagaland becomes first north-eastern state to launch 91. The World Economic Forum annual meet began in Arunachal Pradesh. Pakke Paga Hornbill Festival (PPHF)
framework. single emergenc number 112 . Davos. World leaders from across the globe flocked to is only conservation festival of Arunachal Pradesh.
63. Madagascar president, Andry Rajoelina, has sworn into The Union Home Minister Rajnath Singh has launched the ski resort town on the Swiss Alps for five days to 108. India s first-ever all-women political party the
office after taking the oath in Antananarivo before nine the single emergenc number 112 under the Emergenc discuss what s ailing the world amid fears of the National Women s Part (NWP) was launched in
judges of the High Constitutional Court. He won nearly Response Support System (ERSS) for the state of global economy sleepwalking into a crisis. Mumbai and it will contest half 283 of the 545 Lok
56% to defeat another former president, Marc Nagaland. 92. Explanation: Ace Indian Shuttler Saina Nehwal was Sabha seats in the 2019 elections.
Ravalomanana. 78. Former Defence Minister George Fernandes (88-years) crowned champion of the Indonesia Masters 2019 109. Explanation: Pacer Mohammed Shami became the
passed away in New Delhi. In 1976, was arrested and fastest Indian to claim 100 wickets in ODIs, reaching

©LegalEdge Tutorials Page 53 of 60 ©LegalEdge Tutorials Page 54 of 60


Replication or other unauthorized use of this material is prohibited by the copyright laws of India Replication or other unauthorized use of this material is prohibited by the copyright laws of India
LE Prep Assist LE Prep Assist
the mark in the opening match against New Zealand honor of Chef de Mission for the Olympics. Baishya is Amazon to ensure the global giants pay a fair share of already passed the Bill. The Bill seeks to amend the
in Napier. also one of the Indian Olympic Association Vice taxes on their massive business operations in Europe. National Council for Teacher Education (NCTE) Act,
110. Explanation: Saurashtra has created history in the Presidents. 138. Reserve Bank allowed a one-time restructuring of 1993.
ongoing Ranji Trophy 2018-19. The team went on to 123. Solution: According to the Henley Passport Index, India existing debt up to Rs 25 crore for the companies which 152. The International Kite Festival has been started in
chase 372 in the second Quarter Final against Uttar jumped two positions from 81st in 2018 to 79th in 2019. have defaulted on payment but the loans given to them Ahmedabad.
Pradesh while playing at Bharat Ratna Shri Atal Bihari Japan retained its top spot as the world s most travel- have continued to be classified as standard assets. 153. MP Bezbarauah is the Head of the High-Level
Vajpayee International Cricket Stadium, Lucknow. friendly passport. 139. General Manager, South Central Railway, V K Yadav has Committee for implementation of Clause VI of Assam
Saurashtra went past the record of Assam which had 124. Solution: Liverpool s Mohamed Salah has been named as been appointed the national transporter s senior-most Accord.
chased down 371. the 2018 Confederation of African Football Player of the bureaucrat by the Appointments Committee of the 154. In the United States, Department of Defence, Chief of
111. Explanation: Guatemala has said it is withdrawing Year. For the second consecutive year he has won the Cabinet. Yadav belongs to the 1980 batch of the Indian Staff, Rear Admiral Kevin Sweeney has resigned, a
from a UN-backed anti-corruption commission. award. Railway Service of Electrical Engineers. month after Defence Secretary James Mattis announced
President Jimmy Morales accused the International 125. Solution: Indian Railway Catering and Tourism 140. Qatar has withdrawn its membership from Organization his departure.
Commission Against Impunity in Guatemala, Corporation (IRCT(c), the Railwa s subsidiar , will of the Petroleum Exporting Countries (OPE(c). The 155. Prime Minister of Norway Erna Solberg has arrived in
commonly known as CICIG, of polarising the country offer insurance of up to INR50 lakh for free for air country had sent an official notification to the OPEC New Delhi on a three-day state visit to India. She will
and putting its security at risk. passengers who book their tickets through its portal. expressing its wish to pull out and to focus on its participate in the Indo-Norwegian Business Conference.
112. Explanation: Infosys co-founder and Aadhaar IRCTC has tied up with Bharti-Axa for the offer. liquefied natural gas production in December. 156. The Union Human Resource Development Ministry
architect Nandan Nilekani will head a high level 126. The Parliament of Macedonia has passed the 141. Giriraj Singh is the present Minister of MSME. constituted a task force to look into the issues of suicides
committee asessing digitization of payments. resolution to amend the constitution of the country to 142. Prime Minister of Norway Erna Solberg has arrived in of the students in Jawahar Navodaya Vidyalayas. The
113. Explanation: World Bank President Jim Yong Kim rename it as the Republic of Northern Macedonia. New Delhi on a three-day state visit to India. She was task force has been constituted under the Chairmanship
unexpectedly resigned, more than three years before 127. Gujarat has become the first state to implement the 10 accompanied by senior officials from her government of Psychiatrist, Dr Jitendra Nagpal.
his term ends in 2022, amid differences with the percent reservation for economically weaker sections and a large business delegation. She will participate in 157. Twenty-time Grand Slam champion Federer is the first
Trump administration over climate change and the of the general category. the Indo-Norwegian Business Conference. player to win three Hopman Cups twice with Bencic
need for more development resources. 128. The Indian Space Research Organisation (Isro) is 143. india scripted history after winning their first-ever and once with Martina Hingis, in 2001.
114. Explanation: Malaysia s King Muhammad V setting up a Human Space Flight Centre in Test series in Australia as the fourth Test at the Sydney 158. The Express Group instituted the Ramnath Goenka
abdicated after two years on the throne, the first time Bengaluru, which will support its quest of making Cricket Ground (SCG) concluded in a draw. Excellence in Journalism Awards in 2005 as part of the
a monarch has stepped down before completing their countr s maiden manned space mission a success. 144. Swit erland s Roger Federer and Belinda Bencic centenary year celebrations of its founder, Ramnath
five-year tenure. 129. National Youth Day is celebrated on 12 January being secured victory over German s Angelique Kerber Goenka.
115. Explanation: Senior IPS officer Prabhat Singh has been the birthday of Swami Vivekananda. and Alexander Zverev to win the Hopman Cup for the 159. The Union Cabinet approved a 10% reservation for
appointed as the director general in the national 130. Explanation: Union Minister of Environment, Forest second year in a row. economicall backward upper castes in government
Human Rights Commission. and Climate Change Dr Harsh Vardhan launched the 145. Former bureaucrat M.P. Bezbaruah will head a nine- jobs.
116. Explanation: Japan Aerospace Exploration Agency National Clean Air Programme aiming to reduce toxic member high-level committee constituted by the 160. the government has announced reservation in jobs for
has launched Tok o base startup Star ALE s mini- particulate matter by 2024,taking 2017 as the base Union government to look into the implementation of those belonging to the upper castes and earning less than
satellite aimed to deliver the world s first artificial year. Clause 6 of the Assam accord of 1985 that provides for Rs 8 lakh per year. A bill to amend Article 15 and 16 of
meteor shower. 131. Taiwan President Tsai Ing Wen appointed a new PM constitutional guaranttes to protect the state s the constitution will be moved in Parliament to this
117. Explanation: The Centre has taken a decision to make Su Tseng Chang following the resignation of the indegenous communities. effect.
all new cars meet Euro 6 standards mandatory for incumbent over the recent electoral setback suffered 146. The Cabinet has approved the restructuring of National 161. Option CExplanation: More than five years after
initial launch from April 1,2020, said Union Minister by the ruling Democratic Progressive Party. Health Agency as National Health Authority for better Malayalam was declared a classical language, the
for Transport Nitin Gadkari addressing Symposium 132. Nicolas Maduro was once again sworn in as implementation of Pradhan Mantri Jan Arogya Yojana. Union Government issued its nod for setting up the
of International Automotive Technology 2019 in Pune. Vene uela s President for a second term, from 2019 to 147. The Indian Parliament passed the Right of Children to Centre for Classical Language at the Thunchath
118. Explanation: Defence Minister Smt Nirmala 2025. The swearing-in ceremony took place inside the Free and Compulsory Education (Amendment) Bill, 2018 Ezhuthachan Malayalam University in Tirur.
Sitharaman inaugurated the 426.60 metre long Diffo Supreme Court of Venezuela under Article 231 of the that seeks to do away with the no-detention policy in 162. Pakistan s cabinet has approved the issuance of first-ever
Bridge over Chipu River in Lower Dibang Valley Venezuelan Constitution. schools. The Bill seeks to amend the Right to Education renminbi-denominated Panda Bonds to raise loans from
District in Arunachal Pradesh. 133. President Ram Nath Kovind conferred the honorary (RTE) Act to abolish the no-detention polic in schools. China s capital markets.
119. Explanation: 12 year old Tamil Nadu boy, D Gukesh rank of General of the Indian Army on Purna Chandra 148. U.S. President Donald J. Trump signed into law the Asia 163. Legendary filmmaker Mrinal Sen passed away at the age
became the youngest grandmaster in India and second Thapa,the chief of the Army Staff of Nepalese Army. Reassurance Initiative Act (ARI(a), which passed the U.S. of 95 in Kolkata. He was a Padma Bhushan and Dada
in the world. He achieved this feat by beating D K 134. The Madras High Court has set up an expert committee Senate earlier in December. According to the White Saheb Phalke awardee.
Sharma at 17th Delhi International Open Chess to find a solution to root out invasive species of plants House, the act establishes a multifaceted U.S. strategy to 164. Oslo is the Capital of Norway.
tournament in Delhi. from the Western Ghats. The committee shall be headed increase U.S. security, economic interests, and values in 165. The National Green Tribunal has imposed a fine of 100
120. Explanation: The 38-year-old K Dhanya Sanal, a by Cherukuri Raghavendra Babu, the chairman of the the Indo-Pacific region. crore rupees on Meghalaya government for its failure to
spokesperson for the Union Ministry of Defence, has expert committee on invasive species in Chennai 149. U.S. President Donald J. Trump signed into law the Asia curb illegal coal mining in the state.
become the first woman, who is trekking to National Biodiversity Authority. Reassurance Initiative Act (ARI(a). Specifically, the ARIA 166. The 106th session of Indian Science Congress (IS(c), 2019
Agasthyarkoodam peak, after the Kerala High Court 135. Zoran Zaev is the Prime Minister of Macedonia. will authorize $1.5 billion in spending for a range of U.S. concluded in Lovely Professional University, Jalandhar,
lifted the ban on women s trekking on December 9 last 136. The Reserve Bank of India releases India's Financial programs in East and Southeast Asia and develop a long- Punjab. The five-day session was inaugurated by PM
year. Stability Report (FSR). The FSR reflects the collective term strategic vision. Narendra Modi and attended by Union Science &
121. Solution: Nicolás Maduro was sworn in for a second term assessment of the Sub-Committee of the Financial 150. Arunima Sinha, the first female amputee to have Technology Minister Dr. Harsh Vardhan.
as Vene uela s President amid the economic and Stability and Development Council (FSD(c) on risks to conquered Mt Everest, created yet another record as she 167. The HRD ministry has introduced a 70-point grading
humanitarian crisis faced by the Latin American country. financial stability, as also the resilience of the financial became world s first woman amputee to climb Mount index to assess the quality of school education offered by
122. Exp President of the Indian Weightlifting Federation system. The Global Financial Stability Report (GFSR) is Vinson, the highest peak in Antarctica. states. A state performance grading will be done wherein
(IWF), Birendra Prasad Baishya, has been named Chef de released by the International Monetary Fund (IMF). 151. The Indian Parliament passed the National Council for the states will be marked out of 1,000 scores on 70
Mission for the 2020 Tokyo Olympics. It is for the first 137. France has been pushing hard for a new so-called GAFA Teacher Education (Amendment) Bill, 2018, following parameters.
time that the Weightlifting has been accorded with the tax named after Google, Apple, Facebook and the approval of the Rajya Sabha. The Lok Sabha had

©LegalEdge Tutorials Page 55 of 60 ©LegalEdge Tutorials Page 56 of 60


Replication or other unauthorized use of this material is prohibited by the copyright laws of India Replication or other unauthorized use of this material is prohibited by the copyright laws of India
LE Prep Assist LE Prep Assist
168. The income earned by an average Indian doubled in 7 182. Director General of Ministry of Tourism Shri Satyajeet Pakistan (the judicature branch of government) and the reserves, Uzbekistan is self-sufficient in energy. It is also
years, from Rs 63,642 per year in 2011-12 to Rs 1.25 lakh Rajan has inaugurated the six-day cultural extravaganza chief judge of the Supreme Court of Pakistan. among the world s top 10 gold producers and the fifth
in 2018-19, the first advance estimates of national income Bharat Parv (4th edition) at Red Fort in Delhi, As part of 193. Prime Minister Shri Narendra Modi has inaugurated the largest cotton exporter.
released by the Central Statistics Office (CSO) show. the Republic Day 2019 celebrations. The objective of the countr s first National Museum of Indian Cinema 198. Addis Ababa is the capital of Ethiopia. Birr is the
169. Mala sia s King Muhammad V abdicated after two ears event is to generate a patriotic mood, promote the rich (NMI(c) in Mumbai. The museum has been built by currency of Ethiopia.
on the throne, the first time a monarch has stepped down cultural diversity of the country and to ensure wider NBCC (Indi(a) Limited, at a cost of Rs. 140.61 crore. The Exp Note:
before completing their five-year tenure. participation of the general public. objective of the Museum is to take its visitors through an 1. Yerevan is the capital of Armenia.
170. World Bank President, Jim Yong Kim has made the 183. The annual bilateral maritime exercise KONKAN 18 absorbing journey of over a century of Indian cinema in 2. Minsk is the capital of Belarus.
surprise announcement that he is stepping down after six was held between the Indian Navy and the Royal Navy a story telling mode with the help of visuals, graphics, 3. Phnom Penh is the capital of Cambodia.
years in the post. His resignation will take effect from 1 from Nov 28 to Dec 6, 2018 in Goa. artifacts, interactive exhibits and multimedia expositions. 4. Nicosia is the capital of Cyprus.
February. 184. Ace shuttler Saina Nehwal (Indi(a) has won the 194. The Armed Forces Flag Day is observed annually in India 199. India ranked at 80th positions on the Global Talent
171. Exp Mysore-born Gita Gopinath has joined International Indonesia Masters women s singles title after Carolina on December 7 since 1949. This day is dedicated to India Competitive Index (GTCI) 2019.
Monetary Fund as its chief economist, becoming the first Marin (Spain) limped out of the final due to a leg injury. towards collection of funds from people of India for the The index is released b INSEAD business school in
woman to occupy the top IMF post. The DAIHATSU Indonesia Masters 2019 is a badminton welfare of the Indian Armed Forces Personnel. partnership with Tata Communications and Adecco
172. The government has appointed Sudhir Bhargava as Chief tournament that takes place at the Istora Gelora Bung 195. The Constitution Day has been observed across India on Group.
Information Commissioner (CI(c), along with four Karno in Indonesia and has a total purse of USD 350,000. November 26 to celebrate adoption of the Constitution. China emerged as the best performer among the
information commissioners. Mr. Bhargava was 185. South African President Cyril Ramaphosa was the Chief The Day is being observed jointly by NITI Aayog & Law BRICS countries, with an overall position of 45th.
Information Commissioner in the CIC. Guest at the 2019 Republic Day celebrations in New Commission of India. Swit erland topped the index which is followed by
173. Indian cricketer Ambati Rayudu has been suspended Delhi. Note: Ministry of Human Resources Development has Singapore and the US.
from bowling in International Cricket by the Since 1950, India has been hosting head of state or launched a Digital Signature Campaign on Constitution Last ear in 218, India was ranked at 81st position.
International Cricket Council (IC(c), after having found government of another country as the state guest of Day affirming faith in Indian Constitution and to The report measures levels of Global Talent
his bowling action to be illegal. Earlier, Rayudu had been honour for Republic Day (26 th-January) celebrations in generate awareness on fundamental duties enshrined in Competitiveness by looking at 68 variables such as ease
reported for suspect bowling action during the first ODI New Delhi. the Constitution. of ease of hiring, gender earnings gap, and prevalence
match between India and Australia in Sydney. 186. Prime minister Narendra Modi has inaugurated the 15th 196. Dampa Tiger Reserve or Dampha Tiger Reserve, the of training in firms.
174. Self Explanatory. Pravasi Bharatiya Divas at Varanasi in Uttar Pradesh in largest wildlife sanctuary in Mizoram. 200. On 25th January 2019, Himachal Pradesh celebrated its
175. The Telangana government has decided to form a 'State January 2019. The theme this year is, 'Role of Indian 197. Tashkent is the capital of Uzbekistan. Som is the currency 49th Statehood Day.
Tiger Protection Force' to save the Tiger population in the Diaspora in building New India'. of Uzbekistan. Uzbekistan is a Central Asian nation and On 18 December 1970 the State of Himachal Pradesh
State. The STPF armed (112-member) will be headed by 187. American Pulitzer Prize-winning writer Russell Wayne former Soviet republic. It's known for its mosques, Act was passed by Parliament and the new state came
an Assistant Conservator of Forests to protect the tiger Baker passed away at his home. He was a columnist for mausoleums and other sites linked to the Silk Road, the into being on 25 January 1971 making it the 18th state of
population in Amarabad and Kawal Tiger Reserves. The New York Times from 1962 to 1998 and hosted the ancient trade route between China and the India.
176. The Central Industrial Securit Force has won the Best PBS show Masterpiece Theatre from 1992 to 2004. Mediterranean. With large gas and oil Y.S. Parmar became the state s first chief minister.
Tableue Troph of Republic Da Parade 2019 held at 188. Former president Pranab Mukherjee, Nanaji Deshmukh
Rajpath. The Tableau paid tribute to the Father of the (Bharatiya Jana Sangh leader) and Bhupen Hazarika
Nation Mahatma Gandhi , b highlighting CISF securit (singer) conferred the country's highest civilian award
cover to the Samadhi in the front portion of the Tableau. Bharat Ratna for the year 2019. Pranab Mukherjee
177. Japan's Naomi Osaka has won her maiden Australian (known as 'Pranab Da') was the president between 2012
Open Grand Slam Singles title 2019 at Melbourne, and 2017. Former Prime Minister Atal Bihari Vajpayee
Australia.In the Final, Osaka defeated Czech Petra and founder of Banaras Hindu University Madan Mohan
Kvitova, 7-6, 5-7, 6-4 to become the first Asian player to Malviya were given the award by the Narendra Modi
achieve the world number one ranking. government in 2015.
178. Anders Antonsen (Denmark) has won his first ever 189. Former president Pranab Mukherjee, Nanaji Deshmukh
Indonesia Masters after defeating world number one (Bharatiya Jana Sangh leader) and Bhupen Hazarika
Kento Momota (Japan) to claim the 2019 Indonesia (singer) conferred the country's highest civilian award
Masters title. Bharat Ratna for the year 2019. The Bharat Ratna is the
179. The 'Beating Retreat' ceremony held every year on 29th highest civilian award of the Republic of India.
January, marking the culmination of the four-day-long 190. Manipur, Meghalaya and Tripura celebrated their
Republic Day celebrations. statehood day on 21st January every year.
180. Uttarakhand Government has obtained the permission to On this day in 1972, all the three states became full-
develop the Tulip garden in Pithoragarh district. fledged states under the North Eastern Region
It will be the second Tulip garden (first in Srinagar, J&K) (Reorganisation) Act, 1971.
in the country. Tulips (Tulip (a) flowers are usually large, 191. The theme this year Pravasi Bhartiya Divas is, 'Role of
showy and brightly coloured, generally red, pink, Indian Diaspora in building New India'. Pravasi Bhartiya
yellow, or white (usually in warm colours). Divas is celebrated every year to mark the contribution
181. Sikkim Chief Minister Pawan Kumar Chamling has of the overseas Indian community to the development of
launched the One Famil One Job scheme in Gangtok. India.
Now, Sikkim has become the first state in the country to 192. Justice Asif Saeed Khosa took oath as the 26th Chief
carry out such a programme for the people entitling them Justice of Pakistan. President Arif Alvi administered oath
to state government employee benefits. Under this to the new Chief Justice who took office after the
Scheme, every family one government job has been retirement of Chief Justice Saqib Nisar. The Chief Justice
allotted. of Pakistan (CJP) is the head of the court system of

©LegalEdge Tutorials Page 57 of 60 ©LegalEdge Tutorials Page 58 of 60


Replication or other unauthorized use of this material is prohibited by the copyright laws of India Replication or other unauthorized use of this material is prohibited by the copyright laws of India
LE Prep Assist LE Prep Assist

Notes: Notes:

©LegalEdge Tutorials Page 59 of 60 ©LegalEdge Tutorials Page 60 of 60


Replication or other unauthorized use of this material is prohibited by the copyright laws of India Replication or other unauthorized use of this material is prohibited by the copyright laws of India

S-ar putea să vă placă și